Download as pdf or txt
Download as pdf or txt
You are on page 1of 99

Total Marks : 200

Test-25 (Subject)
( INSTA Prelims Test Series 2021 )

1. Which ddx of the following are RBIs guidelines for universal bank licensing?
1. The requirement of Non-Operative Financial Holding Company (NOFHC) is mandatory for
individual promoters.
2. Not more than 49% of the total paid-up equity capital of the NOFHC shall be owned by the
Promoter/Promoter Group.
3. The bank shall get its shares listed on the stock exchanges within six years of the
commencement of business by the bank.

Which of the statements given above is/are correct?


A. 1 only
B. 2 and 3 only
C. 3 only
D. 1, 2 and 3

Correct Answer : C

Answer Justification :

What is Universal banking?

It is a system of banking where banks undertake a blanket of financial services like


investment banking, commercial banking, development banking, insurance and other financial
services including functions of merchant banking, mutual funds, factoring, housing finance,
insurance etc.

RBIs universal bank licensing guidelines:

1. Individuals/professionals who are ‘residents’ and have minimum 10 years of experience


in banking and finance at a senior level.
2. The initial minimum paid-up voting equity capital for a bank shall be ` five billion.
Thereafter, the bank shall have a minimum net worth of ` five billion at all times.
3. The requirement of Non-Operative Financial Holding Company (NOFHC) is not
mandatory for individual promoters or standalone promoting/converting entities
who/which do not have other group entities.Hence, statement 1 is incorrect.
4. Not less than 51% of the total paid-up equity capital of the NOFHC shall be
owned by the Promoter/Promoter Group. Hence, statement 2 is incorrect. No
shareholder, other than the promoters/promoter group, shall have significant influence
and control in the NOFHC.
5. The bank shall get its shares listed on the stock exchanges within six years of
the commencement of business by the bank.
6. The bank is precluded from having any exposure to its promoters, major shareholders
who have shareholding of 10 per cent or more of paid-up equity shares in the bank, the
relatives of the promoters as also the entities in which they have significant influence or
control.
7. The bank has to open at least 25 per cent of its branches in unbanked rural centres.
8. The bank shall comply with the priority sector lending targets and sub-targets as

www.insightsactivelearn.com 1
Total Marks : 200
Test-25 (Subject)
( INSTA Prelims Test Series 2021 )

applicable to the existing domestic scheduled commercial banks.


9. The board of the bank should have a majority of independent directors.
10. The validity of the in-principle approval issued by the Reserve Bank will be 18 months
from the date of granting in-principle approval and would thereafter lapse automatically.

2. Consider the following statements regarding government debt


1. The central government’s debt as a percentage of gross domestic product (GDP) increased
marginally by 0.1% from fiscal 2017-18 to 45.7% in FY19.
2. External debt stood at 5.4 % of GDP in FY19.

Which of the statements given above is/are correct?


A. 1 only
B. 2 only
C. Both 1 and 2
D. Neither 1 nor 2

Correct Answer : D

Answer Justification :

9th EDITION OF THE STATUS PAPER ON GOVERNMENT DEBT

About:

This document covers details of the financing operations of fiscal deficit of the Central
Government during the year 2018-19. It also contains Debt Management Strategy of the
Central Government for the financial years from 2019-20 to 2021-22.
The Central Government has been bringing-out a Status Paper on Government Debt
since 2010-11.

Key highlights of 9th edition:

The central government’s debt as a percentage of gross domestic product (GDP)


dropped marginally by 0.1% from 45.8% in fiscal 2017-18 to 45.7% or Rs 86.73
lakh crore in FY19.Hence, statement 1 is incorrect.
the general government debt to GDP ratio, which includes the combined debt of the
Centre and states, declined by 0.1% from 68.7% in March 2018 to 68.6% or Rs 1.3 crore
crore (Rs 130 trillion) in March last year.
As external debt stood at 2.7% of GDP or Rs 5.12 lakh crore in FY19.Hence,
statement 2 is incorrect.
Around 94.1% of the Centre’s liabilities consisted of domestic debt in FY19, of which
84.4% or Rs 59.68 lakh crore was made up of marketable securities.
The tenure of the longest security was 37 years.

The average interest cost (AIC) for the Centre remained unchanged over FY18 to FY19 at
7.1%. Thus nominal GDP growth net of AIC was 3.9% in FY19, however, this was within the

www.insightsactivelearn.com 2
Total Marks : 200
Test-25 (Subject)
( INSTA Prelims Test Series 2021 )

limits of sustainable debt parameters.

3. Consider the following statements regarding cess and surcharge


1. A cess imposed by the central government is a tax on tax, levied by the government for a
specific purpose.
2. Surcharge is a charge on any tax, charged on the tax already paid.

Which of the statements given above is/are correct?


A. 1 only
B. 2 only
C. Both 1 and 2
D. Neither 1 nor 2

Correct Answer : C

Answer Justification :

All the above statements are correct.

What is a cess? How it differs from surcharge?

A cess imposed by the central government is a tax on tax, levied by the government
for a specific purpose. Generally, cess is expected to be levied till the time the government
gets enough money for that purpose.

What is surcharge?

Surcharge is a charge on any tax, charged on the tax already paid.As the name
suggests, surcharge is an additional charge or tax. The main surcharges are that on
personal income tax (on high income slabs and on super rich) and on corporate
income tax.

Another major feature of cess like surcharges is that the Centre need not share it with states.
But regarding all other major taxes they come under the divisible pool and hence they shall be
shared with the states with the recommendations of the Finance Commission.

A common feature of both surcharge and cess is that the centre need not share it with states.
Following are the difference between the usual taxes, surcharge and cess.

1. The usual taxes goes to the consolidated fund of India and can be spend for any
purposes.
2. Surcharge also goes to the consolidated fund of India and can be spent for any purposes.
3. Cess goes to Consolidated Fund of India but can be spend only for the specific purposes.

The main difference between surcharge and cess is that despite they are not shareable with
state governments, surcharge can be kept with the CFI and spent like any other taxes, the cess

www.insightsactivelearn.com 3
Total Marks : 200
Test-25 (Subject)
( INSTA Prelims Test Series 2021 )

should be kept as a separate fund after allocating to CFI and can be spent only for a specific
purpose. This means cess can be spent only for the specific purpose for which it is created. If
the purpose for which the cess is created is fulfilled, it should be eliminated.

4. Consider the following statements regarding PM CARES Fund


1. The PM CARES Fund was created with the primary objective of dealing with COVID-19
pandemic only.
2. Prime Minister is the ex-officio Chairman of the PM CARES Fund.

Which of the statements given above is/are correct?


A. 1 only
B. 2 only
C. Both 1 and 2
D. Neither 1 nor 2

Correct Answer : B

Answer Justification :

PM CARES Fund Not A ‘Public Authority’ Under RTI Act

Context: Stating that PM CARES FUNDS is not a ‘public authority’ under Section 2(h) of
the Right to Information Act, 2005, the Prime Minister’s Office(PMO) has refused to
divulge information sought in an application filed under the RTI Act.

What is PM CARES fund?

The PM CARES Fund was created on 28 March 2020, “with the primary objective of
dealing with any kind of emergency or distress situation, like posed by the COVID-19
pandemic”. Hence, statement 1 is incorrect.

It is a “public charitable trust”.

Who administers the fund?

Prime Minister is the ex-officio Chairman of the PM CARES Fund and Minister of
Defence, Minister of Home Affairs and Minister of Finance, Government of India are ex-officio
Trustees of the Fund.

Why it should be monitored?

Reports suggest that PM CARES has already received contributions in excess of Rs.
10,000 crores. It amassed a staggering Rs. 6,500 crores in its very first week with donations
from large corporate houses and celebrities.

What’s government ‘s argument?

www.insightsactivelearn.com 4
Total Marks : 200
Test-25 (Subject)
( INSTA Prelims Test Series 2021 )

The argument against conferring PM CARES the status of a “public account” seems to be that
it is a fund based on voluntary contributions of individuals and organizations, and as such,
beyond the full-fledged scrutiny of the CAG.

Need of the hour:

The very purpose of having a separate public account of India under Article 266(2), as
against the Consolidated Fund of India [Article 266(1)] and the Contingency Fund of
India (Article 267), is to cover receipts that do not fall in either of these two funds.

Similarly, since PM CARES conforms to being a “public account” and as vast sums of money
have been collected manifestly at the behest of the government of India, allowing the CAG to
audit it will be a step in the direction of transparency and instill public confidence in
the Fund.

What the Constitution of India says?

Under Article 266(2) of the Constitution, “public moneys received by or on behalf of the
Government of India”, which is not on account of revenue from taxes, duties, repayment of
loans and the like should be credited to the Public Account of India.

5. Vivad Se Vishwas scheme was initiated to

A. To provide quick loans to stressed MSMEs suffering from COVID pandemic.


B. To provide moratorium on interests on debt less than 2 crores.
C. To provide tax rebate to exporters suffering of COVID pandemic.
D. To settle the huge number of pending direct tax cases.

Correct Answer : D

Answer Justification :

Vivad Se Vishwas scheme

Context: The vivad se vishwas scheme was announced by Union Finance Minister Nirmala
Sitharaman during her budget speech on February 1, 2020. The scheme aims to settle the
huge number of pending direct tax cases. Hence, option (d) is correct.

About Vivad Se Vishwas Scheme: The Direct Tax Vivad Se Vishwas Bill, 2020:

1. The amnesty scheme, at present, covers disputes pending at the level of commissioner
(appeals), Income Tax Appellate Tribunals (ITAT), high courts, the Supreme
Court and those in international arbitration.
2. It offers a complete waiver on interest and penalty to the taxpayers who pay their
pending taxes by March 31.

www.insightsactivelearn.com 5
Total Marks : 200
Test-25 (Subject)
( INSTA Prelims Test Series 2021 )

3. The scheme aims to benefit those whose tax demands are locked in dispute in
multiple forums.
4. If a taxpayer is not able to pay direct taxes by March 31st then, he will get further time
till June 30th. However, in that case, he would have to pay 10 percent more on the tax.

How much?

1. In case it is just the interest and the penalty which is in dispute, the taxpayer will have to
pay 25% of the disputed amount till March 31, and subsequently, it will be 30%.
2. If a taxpayer is not able to pay within the March 31 deadline, he gets a further time till
June 30, but in that case, he would have to pay 10% more on the tax.
3. In case it is just the interest and the penalty which is in dispute, the taxpayer will have to
pay 25% of the disputed amount till March 31, and subsequently, it will be 30%.

Significance:

The scheme aims to resolve 483,000 direct tax-related disputes pending in various appellate
forums.

6. Consider the following statements regarding National Anti-profiteering Authority


1. It is the statutory mechanism under GST law to check the unfair profiteering activities by the
registered suppliers under GST law.
2. The Minister of Finance is the ex-officio chairman.

Which of the statements given above is/are correct?


A. 1 only
B. 2 only
C. Both 1 and 2
D. Neither 1 nor 2

Correct Answer : A

Answer Justification :

National Anti-profiteering Authority

Any reduction in rate of tax on any supply of goods or services or the benefit of input tax credit
should be passed on to the recipient by way of commensurate reduction in prices. However, it
has been the experience of many countries that when GST was introduced there has been a
marked increase in inflation and the prices of the commodities. This happened in spite of the
availability of the tax credit right from the production stage to the final consumption stage
which should have actually reduced the final prices. This was happening because the suppliers
were not passing on the commensurate benefits to the consumer and thereby indulging in
illegal profiteering. National Anti-profiteering Authority (NAA) is therefore being
constituted by the Central Government to examine whether additional input tax

www.insightsactivelearn.com 6
Total Marks : 200
Test-25 (Subject)
( INSTA Prelims Test Series 2021 )

credits availed by any registered person or the reduction in the tax rate have actually
resulted in a commensurate reduction in prices to the recipients.

The National Anti-profiteering Authority comprises of a five-member committee,


consisting of, a Chairman (equivalent to a rank of secretary in the government), four
technical members (present/former commissioners of State tax or central tax departments).
The Additional Director General of Safeguards under the Central Board of Indirect Taxes and
Customs (CBEC) would be the Secretary to NAA. Hence, statement 2 is incorrect.

Vision and Mission of NAA

The National Anti-profiteering Authority (NAA) is the statutory mechanism under


GST law to check the unfair profiteering activities by the registered suppliers under
GST law. The Authority’s core function is to ensure that the commensurate benefits of the
reduction in GST rates on goods and services done by the GST Council and of the Input tax
credit are passed on to the recipients by way of commensurate reduction in the prices by the
suppliers.

7. Consider the following statements regarding eWay Bill


1. EWay Bill is an Electronic Way bill for movement of goods to be generated on the eWay Bill
Portal.
2. A GST registered person cannot transport goods in a vehicle whose value exceeds Rs. 5,00,000
without an e-way bill.

Which of the statements given above is/are correct?


A. 1 only
B. 2 only
C. Both 1 and 2
D. Neither 1 nor 2

Correct Answer : A

Answer Justification :

What is an eWay Bill?


EWay Bill is an Electronic Way bill for movement of goods to be generated on the
eWay Bill Portal. A GST registered person cannot transport goods in a vehicle whose
value exceeds Rs. 50,000 (Single Invoice/bill/delivery challan) without an e-way bill
that is generated on ewaybillgst.gov.in. Hence, statement 2 is incorrect.

Alternatively, Eway bill can also be generated or cancelled through SMS, Android App and by
site-to-site integration through API.
When an eway bill is generated, a unique Eway Bill Number (EBN) is allocated and is available
to the supplier, recipient, and the transporter.

www.insightsactivelearn.com 7
Total Marks : 200
Test-25 (Subject)
( INSTA Prelims Test Series 2021 )

2.When Should eWay Bill be issued?

eWay bill will be generated when there is a movement of goods in a vehicle/ conveyance of
value more than Rs. 50,000 (either each Invoice or in aggregate of all invoices in a
vehicle/conveyance)–

In relation to a ‘supply’
For reasons other than a ‘supply’ (say a return)
Due to inward ‘supply’ from an unregistered person

For this purpose, a supply may be either of the following:

A supply made for a consideration (payment) in the course of business


A supply made for a consideration (payment) which may not be in the course of business
A supply without consideration (without payment) In simpler terms, the term ‘supply’
usually means a:
1. Sale – sale of goods and payment made
2. Transfer – branch transfers for instance
3. Barter/Exchange – where the payment is by goods instead of in money

8. Consider the following statements regarding National Institute of Public Finance and Policy (NIPFP)
1. The institute was a joint initiative of the Ministry of Finance, Planning Commission, several
State governments and distinguished academicians.
2. It is registered under the Societies Registration Act, 1860.
3. It is a center for research in public economics and policies.

Which of the statements given above is/are correct?


A. 1 only
B. 2 and 3 only
C. 3 only
D. 1, 2 and 3

Correct Answer : D

Answer Justification :

All the above statements are correct.

The National Institute of Public Finance and Policy (NIPFP)is a centre for research in
public economics and policies. Founded in 1976, the institute undertakes research, policy
advocacy and capacity building in areas related to public economics. One of the major
mandates of the institute is to assist the Central, State and Local governments in formulating
and reforming public policies by providing an analytical base. The institute was set up as an
autonomous society, at the joint initiative of the Ministry of Finance, Planning
Commission, several State governments and distinguished academicians. It is
registered under the Societies Registration Act, 1860.

www.insightsactivelearn.com 8
Total Marks : 200
Test-25 (Subject)
( INSTA Prelims Test Series 2021 )

In its 40 years of existence, the institute has emerged as a premier think tank in India, and has
made significant contribution to policy reforms at all levels of the government. It has
maintained close functional links with the Central and State governments all along, and has
built up linkages with other teaching and research institutions both in India and abroad.
Although the institute receives an annual grant from the Ministry of Finance, Government of
India, and various State governments, it maintains an independent non-government character
in its pursuit of research and policy.

9. Consider the following statements regarding Urban Co-operative Banks (UCBs)


1. These are allowed to lend money only for non-agricultural purposes.
2. The registration, management, audit and liquidation, etc. governed by RBI under Banking
Regulation Act 1949.

Which of the statements given above is/are correct?


A. 1 only
B. 2 only
C. Both 1 and 2
D. Neither 1 nor 2

Correct Answer : D

Answer Justification :

The term Urban Co-operative Banks (UCBs), though not formally defined, refers to primary
cooperative banks located in urban and semi-urban areas. These banks, till 1996, were
allowed to lend money only for non-agricultural purposes.This distinction does not
hold today. These banks were traditionally centred around communities, localities
work place groups. They essentially lent to small borrowers and businesses. Today,
their scope of operations has widened considerably. Hence, statement 1 is incorrect.

Duality of Control

Concerns regarding the professionalism of urban cooperative banks gave rise to the view that
they should be better regulated. Large cooperative banks with paid-up share capital and
reserves of Rs.1 lakh were brought under the perview of the Banking Regulation Act 1949 with
effect from 1st March, 1966 and within the ambit of the Reserve Bank"s supervision. This
marked the beginning of an era of duality of control over these banks. Banking related
functions (viz. licensing, area of operations, interest rates etc.) were to be governed
by RBI and registration, management, audit and liquidation, etc. governed by State
Governments as per the provisions of respective State Acts. In 1968, UCBS were
extended the benefits of Deposit Insurance. Hence, statement 2 is incorrect.

10. Consider the following statements regarding Regional Rural Banks (RRBs)

www.insightsactivelearn.com 9
Total Marks : 200
Test-25 (Subject)
( INSTA Prelims Test Series 2021 )

1. They were set up on the recommendations of the Narasimham Working Group (1975).
2. The first Regional Rural Bank in India was Prathama Grameen Bank.

Which of the statements given above is/are correct?


A. 1 only
B. 2 only
C. Both 1 and 2
D. Neither 1 nor 2

Correct Answer : C

Answer Justification :

All the above statements are correct.

Regional Rural Banks

The Regional Rural Banks (RRBs) were established in 1975 under the provisions of the
Ordinance promulgated on 26th September, 1975 and Regional Rural Banks Act, 1976 with a
view to developing the rural economy by providing, for the purpose of development of
agriculture, trade, commerce, industry and other productive activities in the rural areas, credit
and other facilities, particularly to small and marginal farmers, agricultural labourers, artisans
and small entrepreneurs, and for matters connected therewith and incidental thereto.

What are RRBs?

Regional Rural Banks were set up on the basis of the recommendations of the Narasimham
Working Group (1975), and after the legislation of the Regional Rural Banks Act, 1976.

The first Regional Rural Bank “Prathama Grameen Bank” was set up on 2nd October,
1975.

The equity of a regional rural bank is held by the Central Government, concerned State
Government and the Sponsor Bank in the proportion of 50:15:35.

11. “Food, work & productivity” was the main objective of which five-year plan?

A. Fifth Plan
B. Sixth Plan
C. Seventh Plan
D. Eighth Plan

Correct Answer : C

www.insightsactivelearn.com 10
Total Marks : 200
Test-25 (Subject)
( INSTA Prelims Test Series 2021 )

Answer Justification :

Seventh Plan

(1985 - 90)

The Plan aimed at accelerating food grain production, increasing employment opportunities &
raising productivity with focus on ‘food, work & productivity’.

The plan was very successful as the economy recorded 6% growth rate against the targeted
5% with the decade of 80’s struggling out of the’ Hindu Rate of Growth’. Hence, option (c)
is correct.

12. Consider the following statements regarding John Maynard Keynes


1. The General Theory of Employment, Interest and Money was published by John Maynard
Keynes.
2. Keynes suggested strong government intervention in the economy.
3. He opined that the invisible hand brings equilibirium to the economy but by ‘strangulating the
poor’.

Which of the statements given above is/are correct?


A. 1 only
B. 2 and 3 only
C. 3 only
D. 1, 2 and 3

Correct Answer : D

Answer Justification :

Mixed Economy

The belief in the self-correcting quality of the market and the ‘invisible hand’ of Adam Smith
got a major setback in early 20th century during the Great Depression (1929). The impact of
the depression spread from the USA to other economies of Western Europe escalating large
scale unemployment, downfall in demand and economic activities and lockouts in industrial
enterprises. The prevailing Smithonian macro ideas failed to check the crisis. A new approach
was needed which came in the famous work, TheGeneral Theory of Employment, Interest and
Money (1936) by the English economist at Cambridge University, John Maynard Keynes
(1883–1946). Keynes questioned the very principles of ‘laissez-faire’ and the nature of the
‘invisible hand’. He even opined that the invisible hand brings equilibirium to the
economy but by ‘strangulating the poor’. He suggested that prices and wages are not
flexible enough to provide employment to all. It means there will be some people unemployed
when the economy will be at its full potential. Ultimately, a fall in demand will be imminent
resulting in recession and if unchecked, in depression which happened in 1929. Questioning

www.insightsactivelearn.com 11
Total Marks : 200
Test-25 (Subject)
( INSTA Prelims Test Series 2021 )

the limitations of the market mechanism, Keynes suggested strong


governmentintervention in the economy. To get the economy out of the depression, he
suggested an increase in government expenditures, discretionary fiscal policy (fiscal deficit,
lower interest rates, cheap money supply, etc.) to boost the demand of goods and services as
this was the reason behind the depression. As Keynesian policies were followed, the concerned
economies were successfully pulled out of the Great Depression.

All the above statements are correct.

13. Consider the following statements regarding Washington consensus


1. The term ‘Washington Consensus’ was coined by the US economist John Williamson.
2. They are a set of policy reforms which most of the official in International Monetary Fund and
World Bank, thought would be good for the crisis-driven Latin American countries of the time.

Which of the statements given above is/are correct?


A. 1 only
B. 2 only
C. Both 1 and 2
D. Neither 1 nor 2

Correct Answer : C

Answer Justification :

All the above statements are correct.

Washington consensus

The term ‘Washington Consensus’ was coined by the US economist John


Williamson(in 1989) under which he had suggesteda set of policyreforms which most
of the official in Washington (i.e., International Monetary Fund and World Bank)
thought would be good for the crisis-driven Latin American countries of the time. The
policy reforms included ten propositions:

(i) Fiscal discipline

(ii) A redirection of public expenditure priorities toward fields offering both high economic
returns and the potentialto improve income distribution, such asprimary health care, primary
education,and infrastructure.

(iii) Tax reform (to lower marginal rates and broaden the tax base)

(iv) Interest rate liberalisation

(v) A competitive exchange rate

www.insightsactivelearn.com 12
Total Marks : 200
Test-25 (Subject)
( INSTA Prelims Test Series 2021 )

(vi) Trade liberalisation

(vii) Liberalisation of FDI inflows

(viii) Privatisation

(ix) Deregulation (in the sense of abolishing barriers to entry and exit)

(x) Secure property rights

However, in coming times, the term became synonymous to neo-liberalism (in Latin America),
market fundamentalism (as George Soros told in 1998) and even globalisation across the
world. It has often been used to describe an extreme and dogmatic commitment to the belief
that marketscan handle everything.

14. Consider the following statements regarding Monetary policy


1. Reserve Bank of India (RBI) is vested with the responsibility of conducting monetary policy
and it is explicitly mandated under the Reserve Bank of India Act, 1934.
2. The Financial Market Committee (FMC) determines the policy interest rate required to
achieve the inflation target.

Which of the statements given above is/are correct?


A. 1 only
B. 2 only
C. Both 1 and 2
D. Neither 1 nor 2

Correct Answer : A

Answer Justification :
Monetary policy refers to the policy of the central bank with regard to the use of
monetary instruments under its control to achieve the goals specified in the Act.
The Reserve Bank of India (RBI) is vested with the responsibility of conducting
monetary policy. This responsibility is explicitly mandated under the Reserve
Bank of India Act, 1934.

The Monetary Policy Process

The Monetary Policy Committee (MPC) constituted by the Central Government under
Section 45ZB determines the policy interest rate required to achieve the inflation
target.

The Reserve Bank’s Monetary Policy Department (MPD) assists the MPC in formulating the
monetary policy. Views of key stakeholders in the economy, and analytical work of the Reserve
Bank contribute to the process for arriving at the decision on the policy repo rate.

www.insightsactivelearn.com 13
Total Marks : 200
Test-25 (Subject)
( INSTA Prelims Test Series 2021 )

The Financial Market Committee (FMC) meets daily to review the liquidity conditions so as
to ensure that the operating target of monetary policy (weighted average lending rate) is kept
close to the policy repo rate. Hence, statement 2 is incorrect.

15. Consider the following statements regarding priority sector lending


1. Total Priority Sector for Domestic commercial banks is 40 per cent of Adjusted Net Bank
Credit (ANBC).
2. Total Priority Sector for Foreign banks with less than 20 branches is 32 per cent of Adjusted
Net Bank Credit (ANBC).

Which of the statements given above is/are correct?


A. 1 only
B. 2 only
C. Both 1 and 2
D. Neither 1 nor 2

Correct Answer : A

Answer Justification :

Adjusted Net Bank Credit (ANBC)

Targets /Sub-targets for Priority sector

The targets and sub-targets set under priority sector lending, to be computed on the basis of
the ANBC/ CEOBE as applicable as on the corresponding date of the preceding year, are as
under:

Domestic
commercial banks
(excl. RRBs & Foreign banks Small
Regional Rural
Categories SFBs) & foreign with less than 20 Finance
Banks
banks with 20 branches Banks
branches and
above

www.insightsactivelearn.com 14
Total Marks : 200
Test-25 (Subject)
( INSTA Prelims Test Series 2021 )

Total 40 per cent of 40 per cent of 75 per cent of 75 per cent of


Priority ANBC as ANBC as ANBC as computed ANBC as
Sector computed in para computed in in para 6 below or computed in
6 below or para 6 below or CEOBE whichever para 6 below
CEOBE CEOBE is higher; or CEOBE
whichever is whichever is However, lending whichever is
higher higher; out of to Medium higher.
which up to 32% Enterprises, Social
can be in the Infrastructure and
form of lending Renewable Energy
to Exports and shall be reckoned
not less than 8% for priority sector
can be to any achievement only
other priority up to 15 per cent
sector of ANBC.
Agriculture 18 per cent of Not applicable 18 per cent ANBC 18 per cent of
ANBC or CEOBE, or CEOBE, ANBC or
whichever is whichever is CEOBE,
higher; out of higher; out of whichever is
which a target of which a target of higher; out of
10 percent# is 10 percent# is which a
prescribed for prescribed for target of 10
Small and SMFs percent# is
Marginal Farmers prescribed
(SMFs) for SMFs
Micro 7.5 per cent of Not applicable 7.5 per cent of 7.5 per cent
Enterprises ANBC or CEOBE, ANBC or CEOBE, of ANBC or
whichever is whichever is CEOBE,
higher higher whichever is
higher
Advances to 12 percent# of Not applicable 15 per cent of 12 percent#
Weaker ANBC or CEOBE, ANBC or CEOBE, of ANBC or
Sections whichever is whichever is CEOBE,
higher higher whichever is
higher

Hence, statement 2 is incorrect.

16. Consider the following statements regarding Rural Infrastructure Development Fund (RIDF)
1. Government of India created the RIDF in Reserve Bank of India (RBI) in 1995-96.
2. Under this, the project for rural connectivity, social and agri-related sector, are eligible for
loans from 80 to 95% of project cost.

Which of the statements given above is/are correct?


A. 1 only
B. 2 only
C. Both 1 and 2

www.insightsactivelearn.com 15
Total Marks : 200
Test-25 (Subject)
( INSTA Prelims Test Series 2021 )

D. Neither 1 nor 2

Correct Answer : B

Answer Justification :

Rural Infrastructure Development Fund

Government of India created the RIDF in NABARD in 1995-96, with an initial corpus of
Rs.2,000 crore. With the allocation of Rs.29,848 crore for 2020-21 under RIDF XXVI, the
cumulative allocation has reached Rs.3,78,348 crore, including Rs. 18,500 crore under Bharat
Nirman. Hence, statement 1 is incorrect.

Eligible Activities

At present, there are 37 eligible activities under RIDF as approved by GoI. (Annexure I). The
eligible activities are classified under three broad categories i.e.

• Agriculture and related sector

• Social sector

• Rural connectivity

Annexure I - Eligible Activities

Eligible Institutions

State Governments / Union Territories


State Owned Corporations / State Govt. Undertakings
State Govt. Sponsored / Supported Organisations
Panchayat Raj Institutions/Self Help Groups (SHGs)/ NGOs
{provided the projects are submitted through the nodal department of State Government
(i.e Finance Department) }

Mode of Finance

NABARD releases the sanctioned amount on reimbursement basis except for the initial
mobilisation advance @30% to North Eastern & Hilly States and 20% for other states.

Quantum of Loan and Margin/Borrower Contribution

The project for rural connectivity, social and agri-related sector, are eligible for loans
from 80 to 95% of project cost. Cost escalation proposals for certain genuine reasons are
considered within two years of sanction.

17. Consider the following statements regarding Small Finance Banks

www.insightsactivelearn.com 16
Total Marks : 200
Test-25 (Subject)
( INSTA Prelims Test Series 2021 )

1. Companies and societies owned and controlled by residents will be eligible to set up small
finance banks.
2. Existing Non-Banking Finance Companies (NBFCs), Micro Finance Institutions (MFIs), and
Local Area Banks (LABs) cannot opt for conversion into small finance banks.

Which of the statements given above is/are correct?


A. 1 only
B. 2 only
C. Both 1 and 2
D. Neither 1 nor 2

Correct Answer : A

Answer Justification :

Small Finance Banks

Eligible promoters

Resident individuals/professionals with 10 years of experience in banking and finance;


Companies and societies owned and controlled by residents will be eligible to
set up small finance banks.
Existing Non-Banking Finance Companies (NBFCs), Micro Finance Institutions
(MFIs), and Local Area Banks (LABs) that are owned and controlled by residents
can also opt for conversion into small finance banks. Hence, statement 2 is
incorrect.

Promoter/promoter groups should be ‘fit and proper’ with a sound track record of professional
experience or of running their businesses for at least a period of five years in order to be
eligible to promote small finance banks.

Scope of activities

1. The small finance bank shall primarily undertake basic banking activities of acceptance
of deposits and lending to unserved and underserved sections including small business
units, small and marginal farmers, micro and small industries and unorganised sector
entities.
2. There will not be any restriction in the area of operations of small finance banks.

Capital requirement

The minimum paid-up equity capital for small finance banks shall be Rs. 100 crore.

18. Consider the following statements regarding Kisan Credit Card Scheme
1. KCC can withdraw money from KCC accounts through ATMs but they cannot make payments

www.insightsactivelearn.com 17
Total Marks : 200
Test-25 (Subject)
( INSTA Prelims Test Series 2021 )

through PoS terminals.


2. Under this scheme, collateral security is waived for a loan limit of up to Rs. 1 lakhs.

Which of the statements given above is/are correct?


A. 1 only
B. 2 only
C. Both 1 and 2
D. Neither 1 nor 2

Correct Answer : B

Answer Justification :

Kisan Credit Card Scheme

The Kisan Credit Card Scheme aims at providing timely and adequate credit to farmers to
meet their needs at the time of crop production (cultivation expenses), besides meeting
contingency expenses. It also covers expenses related to ancillary activities through simplified
procedures in obtaining loans as and when needed.

Features of KCC

KCC borrowers shall be issued an ATM-cum -Debit card to enable them to


withdraw money from KCC accounts through ATMs and make payments through
PoS terminals.Hence, statement 1 is incorrect.
The KCC will be in the nature of a revolving account. Credit balance in the account, if
any, will fetch interest at the savings bank rate.
Collateral security is waived for a loan limit of up to Rs. 1 lakh.

Utility of KCC

To meet the short-term credit requirements for cultivation of crops.


To meet Post-harvest expenses.
Loan for marketing of farm produce.
Working capital for maintenance of farm assets and activities allied to agriculture, like
dairy animals, inland fishery, etc.
Consumption requirements of farmer households.
Investment credit requirement for agriculture and allied activities like pump sets,
sprayers, dairy animals, etc.

While 10 per cent of the short-term limit under KCC can be used for household consumption
purposes, it is advisable not to divert more funds for consumption expenditure. The purpose of
KCC is to meet your crop loan requirements. Large amounts spent on household expenses
from KCC funds will affect your capacity to generate income. The income from your farm
activities helps you repay the outstanding loans under KCC. When you divert funds away from
crop activities, you may not be in a position to repay your loan.

www.insightsactivelearn.com 18
Total Marks : 200
Test-25 (Subject)
( INSTA Prelims Test Series 2021 )

19. Consider the following statements regarding National Centre for Financial Education (NCFE)
1. It aims to promote financial education across India for all sections of the population.
2. It is a not-for-profit company promoted by Indian Banks' Association.

Which of the statements given above is/are correct?


A. 1 only
B. 2 only
C. Both 1 and 2
D. Neither 1 nor 2

Correct Answer : A

Answer Justification :

The National Centre for Financial Education (NCFE) is a Section 8 (Not for Profit)
Company promoted by Reserve Bank of India (RBI), Securities and Exchange Board of
India (SEBI), Insurance Regulatory and Development Authority of India (IRDAI) and
Pension Fund Regulatory and Development Authority (PFRDA). Hence, statement 2 is
incorrect.

About NCFE

The National Centre for Financial Education (NCFE)aims to promote financial


education across India for all sections of the population as per the National Strategy
for Financial Education of Financial Stability and Development Council. NCFE creates
financial awareness and empowerment through financial education campaigns across the
country for all sections of the population through seminars, workshops, conclaves, trainings,
programmes, campaigns, discussion forums with/without fees by itself or with help of
institutions, organisations and provide training in financial education and create financial
education material in electronic or non-electronic formats, workbooks, worksheets, literature,
pamphlets, booklets, flyers, technical aids and to prepare appropriate financial literature for
target-based audience on financial markets and financial digital modes for improving financial
literacy so as to improve their knowledge, understanding, skills and competence in finance.

20. Consider the following statements regarding Financial Education Training Programme (FETP)
1. It is an initiative for providing unbiased personal financial education to organizations for
improving financial literacy in the country.
2. FETP is being conducted for school teachers of classes VIII to X across India.

Which of the statements given above is/are correct?


A. 1 only
B. 2 only
C. Both 1 and 2
D. Neither 1 nor 2

www.insightsactivelearn.com 19
Total Marks : 200
Test-25 (Subject)
( INSTA Prelims Test Series 2021 )

Correct Answer : C

Answer Justification :

All the above statements are correct.

Financial Education Programme for Adults (FEPA)

The objective of the Financial Education Programme for Adults (FEPA) is to create financial
awareness that will generate confidence in the financially excluded sections of the society to
use the financial services and products more effectively thereby bringing more people to the
formal financial sector.

The target groups include the adult population such as employees of various organisations,
SHG members, farmers and rural folks, women groups, household people, MGNREGA
cardholders, personnel from forces or any other classes of financially excluded section of the
society. Workshop shall be conducted free of cost and no money will be collected from the
participants. NCFE will provide material free of cost.

Financial Education Training Programme (FETP)

The Financial Education Training Programme (FETP) is an initiative of the NCFE for
providing unbiased personal financial education to people and organizations for
improving financial literacy in the country. The program, based on two pillars;
education and awareness, aims to establish a sustainable financial literacy campaign
that can empower people’s lives.

NCFE is conducting FETP for school teachers of classes VIII to X across India. After
completion of the training, these teachers will be certified as “Money Smart Teacher” and
would facilitate conducting financial education classes in schools and encourage students to
obtain basic financial skills.

21. Consider the following statements regarding RTGS System


1. It is an electronic fund transfer system that operates on a Deferred Net Settlement (DNS)
basis which settles transactions in batches.
2. The RTGS system is primarily meant for large value transactions.

Which of the statements given above is/are correct?


A. 1 only
B. 2 only
C. Both 1 and 2
D. Neither 1 nor 2

Correct Answer : B

www.insightsactivelearn.com 20
Total Marks : 200
Test-25 (Subject)
( INSTA Prelims Test Series 2021 )

Answer Justification :

What is RTGS System?

The acronym 'RTGS' stands for Real Time Gross Settlement, which can be defined as
the continuous (real-time) settlement of funds transfers individually on an order-by-
order basis (without netting). 'Real Time' means the processing of instructions at the
time they are received rather than at some later time; 'Gross Settlement' means the
settlement of funds transfer instructions occurs individually (on an instruction-by-
instruction basis). Considering that the funds settlement takes place in the books of
the Reserve Bank of India, the payments are final and irrevocable. Hence, statement
1 is incorrect.

How RTGS is different from National Electronics Funds Transfer System (NEFT)?

RTGS is meant for large-value instantaneous fund transfers. It happens on a real-time


basis. Every transaction is settled individually in case of RTGS. NEFT is an electronic fund
transfer system that operates on a Deferred Net Settlement (DNS) basis which settles
transactions in batches. Hence, statement 1 is incorrect.

In DNS, the settlement takes place with all transactions received till the particular cut-off
time. These transactions are netted (payable and receivables) in NEFT whereas in RTGS the
transactions are settled individually.

Is there any minimum / maximum amount stipulation for RTGS transactions?

The RTGS system is primarily meant for large value transactions. The minimum amount
to be remitted through RTGS is Rs 2 lakh. There is no upper ceiling for RTGS transactions.

What is the time taken for effecting funds transfer from one account to another under RTGS?

Under normal circumstances the beneficiary branches are expected to receive the funds in
real time as soon as funds are transferred by the remitting bank. The beneficiary bank has to
credit the beneficiary's account within 30 minutes of receiving the funds transfer message.

22. Consider the following statements regarding Local Area Bank Scheme
1. The promoters of a LAB could be individuals, corporate entities and societies but not NRI’s.
2. Individual shareholder voting rights were to be restricted to a ceiling of 10% of the total voting
rights.

Which of the statements given above is/are correct?


A. 1 only
B. 2 only
C. Both 1 and 2
D. Neither 1 nor 2

www.insightsactivelearn.com 21
Total Marks : 200
Test-25 (Subject)
( INSTA Prelims Test Series 2021 )

Correct Answer : B

Answer Justification :

The Local Area Bank Scheme was introduced in August 1996 pursuant to the announcement of
the then Finance Minister. In his budget speech, the Finance Minister referred to the setting
up of new private local banks with jurisdiction over two or three contiguous districts. He
observed that this would enable the mobilization of rural savings by local institutions and
make them available for investments in the local areas. The Local Area Banks (LABs) were
expected to bridge the gaps in credit availability and strengthen the institutional credit
framework in the rural and semi-urban areas.

The background of LAB scheme

1.1 Objective:

In 1996 it was decided to allow the establishment of local banks in the private sector. These
banks were expected to bridge the gaps in credit availability and enhance the institutional
credit framework in the rural and semi-urban areas and provide efficient and competitive
financial intermediation services in their area of operation.

1.2 Capital:

The minimum start-up capital of a LAB was fixed at Rs.5 crore. The promoters of these banks
were required to bring in the entire minimum share capital up-front. It was also decided that a
family among the promoter group could hold equity not exceeding 40% of the capital. The NRI
contributions to the equity of the bank were not to exceed 40% of the paid-up capital. The
entire initial capital subscribed by the promoters (including their friends and
relatives/associates) would carry a lock in period of three years from the date of licensing of
the bank. Further, the promoters’ equity to the extent of 40% of the initial paid- up capital was
to be locked in at least for two years beyond the aforesaid period of three years subject to
review before expiry of five years from the date of licensing of the bank.

1.3 Promoters:

The promoters of a LAB could be individuals, corporate entities and societies.The


number of NRI promoters was not to exceed 20% of the total number of promoters.
Hence, statement 1 is incorrect.

1.4. Voting Rights:

Individual shareholder voting rights were to be restricted to a ceiling of 10% of the


total voting rightsin terms of Section 12(2) of the Banking Regulation Act, 1949.

1.5 Area of operation:

The area of operation of an LAB was restricted to a maximum of three geographically


contiguous districts. The activities of an LAB were to be focused on local customers
predominantly in rural and semi-urban areas so as to bridge the credit gap in these areas.

www.insightsactivelearn.com 22
Total Marks : 200
Test-25 (Subject)
( INSTA Prelims Test Series 2021 )

23. Consider the following statements regarding Sovereign Gold Bond (SGB) Scheme
1. The Bonds will be denominated in multiples of gram(s) of gold with a basic unit of 1 gram.
2. The tenor of the Bond will be for a period of 5 years with exit option.

Which of the statements given above is/are correct?


A. 1 only
B. 2 only
C. Both 1 and 2
D. Neither 1 nor 2

Correct Answer : A

Answer Justification :

Sovereign Gold Bond (SGB) Scheme

Sl.
Item Details
No.
1 Product name Sovereign Gold Bond 2020-21 (Series IX)
To be issued by Reserve Bank India on behalf of the Government of
2 Issuance
India.
The Bonds will be restricted for sale to resident Indian entities
3 Eligibility including individuals, HUFs, Trusts, Universities and Charitable
Institutions.
The Bonds will be denominated in multiples of gram(s) of gold
4 Denomination
with a basic unit of 1 gram.
The tenor of the Bond will be for a period of 8 years with exit
5 Tenor option after 5th year to be exercised on the interest payment
dates.
6 Minimum size Minimum permissible investment will be 1 gram of gold.
The maximum limit of subscribed shall be 4 KG for individual, 4 Kg for
HUF and 20 Kg for trusts and similar entities per fiscal (April-March)
notified by the Government from time to time. A self-declaration to
7 Maximum limit
this effect will be obtained. The annual ceiling will include bonds
subscribed under different tranches during initial issuance by
Government and those purchase from the Secondary Market.
In case of joint holding, the investment limit of 4 KG will be applied to
8 Joint holder
the first applicant only.
Price of Bond will be fixed in Indian Rupees on the basis of simple
average of closing price of gold of 999 purity published by the India
Bullion and Jewellers Association Limited for the last 3 working days
9 Issue price of the week preceding the subscription period. The issue price of the
Gold Bonds will be Rs 50 per gram less for those who subscribe
online and pay through digital mode. The issue price of the Bond
during the subscription period shall be Rs 5,000 per gram.

www.insightsactivelearn.com 23
Total Marks : 200
Test-25 (Subject)
( INSTA Prelims Test Series 2021 )

Payment for the Bonds will be through cash payment (upto a maximum
10 Payment option
of Rs 20,000) or demand draft or cheque or electronic banking.

Hence, statement 2 is incorrect.

24. Consider the following statements regarding Indian Gold Coin


1. The Indian Gold Coin is part of Indian Gold monetization program.
2. The coin will be the first ever national gold coin will have the national emblem of Ashok
Chakra engraved on one side and the face of Mahatma Gandhi on the other side.
3. The Indian gold coin will be of 24 karat purity and 999 fineness.

Which of the statements given above is/are correct?


A. 1 only
B. 2 and 3 only
C. 3 only
D. 1, 2 and 3

Correct Answer : D

Answer Justification :

All the above statements are correct.

Indian Gold Coin

The Indian Gold Coin is part of Indian Gold monetisation program.The coin will be
the first ever national gold coin will have the national emblem of Ashok Chakra
engraved on one side and the face of Mahatma Gandhi on the other side. Initially, the
coins will be available in denominations of 5 and 10 grams. A 20 grams bar/bullion will also be
available. The Indian Gold Coin is unique in many respects and will carry advanced anti-
counterfeit features and tamper-proof packaging that will aid easy recycling.

Coin features

Purity - The Indian gold coin will be of 24 karat purity and 999 fineness.
Hallmarked - All coins will be hallmarked as per the BIS standards.
Security - The tamperproof packaging and advanced anti-counterfeit features on the coin
cover make s it very safe and easily recyclable.
Availability - This coin will distributed through designated & recognised Metals and
Minerals Trading Corporation of India (MMTC) outlets.

25. Consider the following statements regarding currency management


1. Under RBI Act, 1934, the design of banknotes is required to be approved by the Reserve Bank
of India.

www.insightsactivelearn.com 24
Total Marks : 200
Test-25 (Subject)
( INSTA Prelims Test Series 2021 )

2. The responsibility for coinage vests with the Government of India on the basis of the Coinage
Act, 2011.

Which of the statements given above is/are correct?


A. 1 only
B. 2 only
C. Both 1 and 2
D. Neither 1 nor 2

Correct Answer : B

Answer Justification :

What is the role of the Reserve Bank of India in currency management?

The Reserve Bank derives its role in currency management from the Reserve Bank of India
Act, 1934. The Reserve Bank manages currency in India. The Government, on the advice of the
Reserve Bank, decides on various denominations of banknotes to be issued. The Reserve Bank
also co-ordinates with the Government in the designing of banknotes, including the security
features. The Reserve Bank estimates the quantity of banknotes that are likely to be needed
denomination-wise and accordingly, places indent with the various printing presses. The aim
of the Reserve Bank is to provide good quality notes to members of public. Towards this aim,
the banknotes received back from circulation are examined and those fit for circulation are
reissued and the others (soiled and mutilated) are destroyed so as to maintain the quality of
banknotes in circulation.

What is the role of Government of India?

In terms of Section 25 of RBI Act, 1934 the design of banknotes is required to be


approved by the Central Government on the recommendations of the Central Board of
the Reserve Bank of India.Hence, statement 1 is incorrect.

The responsibility for coinage vests with the Government of India on the basis of the
Coinage Act, 2011 as amended from time to time. The Government of India is also
responsible for the designing and minting of coins in various denominations.

Who decides on the figure to be printed on a new note?

The Government of India in consultation with the Reserve Bank of India decides the
design of banknotes.

26. Consider the following statements regarding Agriculture sector


1. Gross Value Added (GVA) in agriculture increased consecutively from a negative 0.2 per cent
in 2014-15 to 2.9 per cent in 2018-19.
2. Women’s participation in agriculture increased to 25.9 per cent in 2015-16 from 11.7 per cent

www.insightsactivelearn.com 25
Total Marks : 200
Test-25 (Subject)
( INSTA Prelims Test Series 2021 )

in 2005-06.

Which of the statements given above is/are correct?


A. 1 only
B. 2 only
C. Both 1 and 2
D. Neither 1 nor 2

Correct Answer : D

Answer Justification :

Agriculture

Agriculture sector in India typically goes through cyclical movement in terms of its growth.

1. Gross Value Added (GVA) in agriculture improved from a negative 0.2 per cent
in 2014-15 to 6.3 per cent in 2016-17 but decelerated to 2.9 per cent in 2018-19.
2. Gross Capital Formation (GCF) in agriculture as percentage of GVA marginally declined
to 15.2 per cent in 2017-18 as compared to 15.6 per cent in 2016-17. The public sector
GCF in agriculture as a percentage of GVA increased to 2.7 per cent in 2016-17 from 2.1
per cent in 2013-14.
3. Women’s participation in agriculture increased to 13.9 per cent in 2015-16 from
11.7 per cent in 2005-06 and their concentration is highest (28 per cent) among small
and marginal farmers.
4. A shift is seen in the number of operational land holdings and area operated by
operational land holdings towards small and marginal farmers.
5. 89% of groundwater extracted is used for irrigation. Hence, focus should shift from land
productivity to ‘irrigation water productivity’. Thrust should be on micro-irrigation to
improve water use efficiency.
6. Fertilizer response ratio has been declining over time. Organic and natural farming
techniques including Zero Budget Natural Farming (ZBNF) can improve both water use
efficiency and soil fertility.
7. Adopting appropriate technologies through Custom Hiring Centers and implementation
of ICT are critical to improve resource-use efficiency among small and marginal farmers.
8. Diversification of livelihoods is critical for inclusive and sustainable development in
agriculture and allied sectors. Policies should focus on
Dairying as India is the largest producer of milk.
Livestock rearing particularly of small ruminants.
Fisheries sector, as India is the second largest producer.

Hence, both statement 1 and 2 are incorrect.

27. Which of the following are the reasons for involuntary financial exclusion?
1. Lack of trust in the system

www.insightsactivelearn.com 26
Total Marks : 200
Test-25 (Subject)
( INSTA Prelims Test Series 2021 )

2. High transaction costs


3. Lack of requisite documents
4. Poor quality of services rendered

Select the correct answer using the code given below:


A. 1, 2 and 3 only
B. 2, 3 and 4 only
C. 1, 2 and 4 only
D. 1, 2, 3 and 4

Correct Answer : D

Answer Justification :

All the above statements are correct.

Financial inclusion is increasingly being recognized as a key driver of economic growth and
poverty alleviation the world over. Access to formal finance can boost job creation, reduce
vulnerability to economic shocks and increase investments in human capital. Without adequate
access to formal financial services, individuals and firms need to rely on their own limited
resources or rely on costly informal sources of finance to meet their financial needs and
pursue growth opportunities. At a macro level, greater financial inclusion can support
sustainable and inclusive socio-economic growth for all.

An inclusive financial system supports stability, integrity and equitable growth. Therefore,
financial exclusion because of several barriers like physical, socio-cultural and psychological,
warrants attention from the policy makers. Some of the key reasons resulting in involuntary
exclusion are:

1. Lack of surplus income


2. Lack of trust in the system
3. Not suitable to customer’s requirements
4. High transaction costs
5. Lack of requisite documents
6. Remoteness of service provider
7. Lack of awareness about the product
8. Poor quality of services rendered

It is also noteworthy to state that, seven of the seventeen United Nations Sustainable
Development Goals (SDG) of 2030 view financial inclusion as a key enabler for achieving
sustainable development worldwide by improving the quality of lives of poor and marginalized
sections of the society.

28. Which of the following are Pre-requisites for Direct Benefit Transfer?
1. Identification of beneficiaries and digitization of beneficiary database

www.insightsactivelearn.com 27
Total Marks : 200
Test-25 (Subject)
( INSTA Prelims Test Series 2021 )

2. Last mile connectivity/service delivery


3. Seeding of Aadhaar in beneficiary database and bank accounts

Which of the statements given above are correct?


A. 1 and 2 only
B. 2 and 3 only
C. 1 and 3 only
D. 1, 2 and 3

Correct Answer : D

Answer Justification :

All the above statements are correct.

Overview of Direct Benefit Transfer

-8
With the aim of reforming Government delivery system by re-engineering the existing process

m
in welfare schemes for simpler and faster flow of information/funds and to ensure accurate
o
l.c
targeting of the beneficiaries, de-duplication and reduction of fraud Direct Benefit Transfer
(DBT) was started on 1st January, 2013.

i 5 @
With the rapid rollout of Aadhaar in the country, it was felt possible to move to a system of

rt
transferring cash benefits directly to the poor.

Pre-requisites for DBT

1.
- a
Identification of beneficiaries and digitisation of beneficiary database
2.
t i
Opening of bank accounts
r
ha
3. Aadhaar enrolment

nB
4. Seeding of Aadhaar in beneficiary database and bank accounts
5. Last mile connectivity/service delivery

www.insightsactivelearn.com 28
Total Marks : 200
Test-25 (Subject)
( INSTA Prelims Test Series 2021 )

29. Consider the following statements regarding Aerospace and Aviation Sector Skill Council (AASSC)
1. It is the apex body in skill development in the emerging Aerospace and Aviation sector.
2. AASSC is one of the various Sector Skill Councils (SSC) which has been formed with the help
of grant from the National Skill Development Corporation (NSDC).
3. The promoters of AASSC are Hindustan Aeronautics Limited (HAL), Bangalore Chamber of
Industries & Commerce (BCIC) and the Society of Indian Aerospace Technologies & Industries
(SIATI).

Which of the statements given above are correct?


A. 1 and 2 only
B. 2 and 3 only
C. 1 and 3 only
D. 1, 2 and 3

Correct Answer : D

Answer Justification :

All the above statements are correct.

Aerospace and Aviation Sector Skill Council (AASSC)

The Aerospace and Aviation Sector Skill Council (AASSC) is the apex body in skill
development in the emerging Aerospace and Aviation sector. AASSC has been formed
under the Government and Industry led initiative to skill the Indian labour force.

About AASSC

The Aerospace and Aviation Sector Skill Council (AASSC) was incorporated as a Section 8
Company (under Companies Act 2013) on 12th September 2014. AASSC is one of the various
Sector Skill Councils (SSC) which has been formed with the help of grant from the
National Skill Development Corporation (NSDC). NSDC provides support to all SSCs
during the initial years of operations in the form of grant. The promoters of AASSC are
Hindustan Aeronautics Limited (HAL), Bangalore Chamber of Industries & Commerce
(BCIC) and the Society of Indian Aerospace Technologies & Industries (SIATI). In
addition to the equity contribution from its promoters, the company has received grant from
National Skill Development Corporation.

30. Consider the following statements regarding Kumhar Sashaktikaran Yojana


1. Kumbhar Sashaktikaran Program is an initiative of the NABARD for empowerment of potter’s
community.
2. Under the program, new technology pottery equipments like the electric Chaak will be
provided.

www.insightsactivelearn.com 29
Total Marks : 200
Test-25 (Subject)
( INSTA Prelims Test Series 2021 )

Which of the statements given above is/are correct?


A. 1 only
B. 2 only
C. Both 1 and 2
D. Neither 1 nor 2

Correct Answer : B

Answer Justification :

Kumhar Sashaktikaran Yojana

Kumbhar Sashaktikaran Program is an initiative of the Khadi and VillageIndustries


Commission (KVIC) for empowerment of potters community in the remotest of
locations in the country. Hence, statement 1 is incorrect.

Target beneficiaries

The program reaches out to the potters in : U.P., M.P., Maharashtra, J&K, Haryana, Rajasthan,
West Bengal, Assam, Gujarat, Tamil Nadu, Odisha, Telangana and Bihar.

Benefits provided

This program provides the following support to potters.

Training for advanced pottery products


Latest, new technology pottery equipments like the electric Chaak
Market linkages and visibility through KVIC exhibitions

Outcomes

Due to the supply of electric chaaks, the potters have reaped the following benefits.

More production with less hours of work.


Less noise and better health benefits
Less power consumption with smooth transition to higher speeds

31. Consider the following statements regarding Software Technology Parks of India
1. It as an autonomous society under the Ministry of Commerce and Industry.
2. STPI’s main objective has been the promotion of software exports from the country.
3. STPI is the nodal agency for the implementation of India BPO Promotion Scheme (IBPS) and
North East BPO Promotion Scheme (NEBPS) under Digital India Initiative.

Which of the statements given above are correct?


A. 1 and 2 only
B. 2 and 3 only

www.insightsactivelearn.com 30
Total Marks : 200
Test-25 (Subject)
( INSTA Prelims Test Series 2021 )

C. 1 and 3 only
D. 1, 2 and 3

Correct Answer : B

Answer Justification :

Software Technology Parks of India was set up in 1991 as an autonomous society


under theMinistry of Electronics and Information Technology (MeitY). Hence, statement 1
is incorrect.

About STPI

STPI’s main objective has been the promotion of software exports from the country.
STPI acts as a single-window in providing services to the software exporters. The services
rendered by STPI for the software exporting community have been statutory services, data
communications services, incubation facilities, training and value-added services. STPI has
played a key developmental role in the promotion of software exports with a special focus on
SMEs and startup units.

STPI Centres

At present, a total of 60 STPI centres/sub-centres are operational across the country, out of
which 51 centres are in Tier II and Tier III cities. STPI is working closely with the respective
state governments/local authorities for the creation of more space, equipped with
state–of–the–art infrastructure facilities, for the development of the software industry and
increasing exports.

Services

The main services rendered by STPI for the IT/ITES/ESDM industry are statutory services,
incubation facilities, data communications services which inter-alia are as under:

IBPS & NEBPS

STPI is the nodal agency for the implementation of India BPO Promotion Scheme (IBPS)
and North East BPO Promotion Scheme (NEBPS) under Digital India Initiative. The
objectives of the schemes are to create around 1.5 lakh job opportunities for the local youths
of smaller towns and to attract investments in the respective regions for all-round
development. These schemes are helping to create the right ecosystem required for the
growth of smaller towns and shall bring prosperity to those locations.

32. Consider the following statements regarding India Brand Equity Foundation (IBEF)
1. It is a Trust established by the Ministry of Corporate Affairs.
2. IBEF works with a network of stakeholders both domestic and international, to promote Brand

www.insightsactivelearn.com 31
Total Marks : 200
Test-25 (Subject)
( INSTA Prelims Test Series 2021 )

India.

Which of the statements given above is/are correct?


A. 1 only
B. 2 only
C. Both 1 and 2
D. Neither 1 nor 2

Correct Answer : B

Answer Justification :

The India Brand Equity Foundation (IBEF) is a Trust established by the Department
of Commerce, Ministry of Commerce and Industry, Government of India. Hence,
statement 1 is incorrect.

About IBEF

The primary objective of India Brand Equity Foundation (IBEF) is to promote and create
international awareness of the Made in India label in markets overseas and to facilitate
dissemination of knowledge of Indian products and services. Towards this objective, IBEF
works closely with stakeholders across government and industry. IBEF works with a
network of stakeholders - domestic and international - to promote Brand India. The
IBEF produces some of the finest research reports, factbooks, insights papers and case studies
across various sectors, businesses and trends in the Indian economy. These publications offer
authoritative information on business partnerships and growth opportunities in India.

The three main pillars of IBEF are:

Export Promotion
Knowledge Centre
Digital Media

Export Promotion

Promoting Indian exports with 360-degree branding and marketing campaigns is one of the
key mandates for IBEF. It runs campaigns for various important sectors and aims at improving
and increasing the perception of Indian products and services.

Brand India Pharma Campaign


Brand India Engineering
Brand India Agri-Products
Brand India Leather
Brand India Textile

33. Consider the following statements regarding Atal Beemit Vyakti Kalyan Yojana

www.insightsactivelearn.com 32
Total Marks : 200
Test-25 (Subject)
( INSTA Prelims Test Series 2021 )

1. Atal Bimit Vyakti Kalyan Yojana is a welfare measure being implemented by the LIC of India.
2. It offers cash compensation to insured persons when they are rendered unemployed.

Which of the statements given above is/are correct?


A. 1 only
B. 2 only
C. Both 1 and 2
D. Neither 1 nor 2

Correct Answer : B

Answer Justification :

Atal Beemit Vyakti Kalyan Yojana

Atal Bimit Vyakti Kalyan Yojana is a welfare measure being implemented by the
Employee's State Insurance (ESI) Corporation.Hence, statement 1 is incorrect.

It offers cash compensation to insured persons when they are rendered unemployed.

The Scheme was introduced w.e.f. 01-07-2018. The scheme is implemented on pilot basis for a
period of two years initially. The scheme has been extended upto 20 June 2021.

Eligibility

Employees covered under Section 2(9) of the ESI Act 1948.


The Insured Person (IP) should have been rendered unemployed during the period the
relief is claimed.
The Insured Person should have been in insurable employment for a minimum period of
two years.
The Insured Person should have contributed not less than 78 days during each of the
preceding four contribution periods.
The contribution in respect of him should have been paid or payable by the employer.
The contingency of the unemployment should not have been as a result of any
punishment for misconduct or superannuation or voluntary retirement.
Aadhar and Bank Account of the Insured Person should be linked with insured person
database.

34. Consider the following statements regarding SARFAESI Act


1. The SARFAESI Act empowers banks and other financial institutions to directly auction
residential or commercial properties that have been pledged with them to recover loans from
borrowers.
2. One of the major features of the Act is that it is applicable to unsecured creditors also.

Which of the statements given above is/are correct?


A. 1 only

www.insightsactivelearn.com 33
Total Marks : 200
Test-25 (Subject)
( INSTA Prelims Test Series 2021 )

B. 2 only
C. Both 1 and 2
D. Neither 1 nor 2

Correct Answer : A

Answer Justification :

What is it?

The SARFAESI Act was passed on December 17, 2002, in order to lay down processes to help
Indian lenders recover their dues quickly. The SARFAESI Act essentially empowers banks
and other financial institutions to directly auction residential or commercial
properties that have been pledged with them to recover loans from borrowers. Before
this Act took effect, financial institutions had to take recourse to civil suits in the courts to
recover their dues, which is a lengthy and time-consuming process.

As per the SARFAESI Act, if a borrower defaults on a loan financed by a bank against
collateral, then the bank gets sweeping powers to recover its dues from the borrower. After
giving a notice period of 60 days, the lender can take possession of the pledged assets of the
borrower, take over the management of such assets, appoint any person to manage them or
ask debtors of the borrower to pay their dues too, with respect to the asset. This recovery
procedure saves banks and financial institutions a lot of time which otherwise would be long
drawn out due to the intervention of courts.

One of the major drawbacks of the Act is that it is not applicable to unsecured
creditors. Hence, statement 2 is incorrect.

This and other drawbacks in the recovery mechanisms were plugged in the Insolvency and
Bankruptcy Code, 2016. ARCs or Asset Reconstruction Companies which buy out distressed
assets are the other alternative that banks use to offload doubtful debt, to ensure more
focussed and efficient resolution, say experts.

35. Consider the following statements regarding Voluntary retention route for foreign portfolio investors
1. It is a new channel of investment available to FPIs to encourage them to invest in debt markets
in India over and above their investments through the regular route.
2. Initially, Investments under this route shall be capped at Rs 20,000 crore for VRR-GOVT and
40,000 crore per annum for VRR-COPR.
3. It is an initiative of SEBI.

Which of the statements given above is/are correct?


A. 1 only
B. 2 and 3 only
C. 3 only
D. 1, 2 and 3

www.insightsactivelearn.com 34
Total Marks : 200
Test-25 (Subject)
( INSTA Prelims Test Series 2021 )

Correct Answer : A

Answer Justification :

Voluntary retention route for foreign portfolio investors

What is VRR?

It is a new channel of investment available to FPIs to encourage them to invest in


debt markets in India over and above their investments through the regular route.
The objective is to attract long-term and stable FPI investments into debt markets while
providing FPIs with operational flexibility to manage their investments.

When was this route proposed?

This new investment route was proposed by the central bank in October 2018 at a
time the rupee was weakening against the dollar very sharply.Hence, statement 3 is
incorrect.

There were also talks of a special NRI bond scheme to attract more dollar funds into the
economy and stabilise the rupee.

How are they different from the regular FPI investments?

Guidelines say that investments through VRR will be free of the macro-prudential and other
regulatory prescriptions applicable to FPI investments in debt markets, provided FPIs
voluntarily commit to retain a required minimum percentage of their investments in India for a
period of their choice. But the minimum retention period shall be three years, or as decided by
RBI.

How much money can an FPI invest through this route?

Investments under this route as of now shall be capped at Rs 40,000 crore for VRR-
GOVTand 35,000 crore per annum for VRR-COPR.Hence, statement 2 is incorrect.

But the limit could be changed from time to time based on macro-prudential considerations
and assessment of investment demand. There will be separate limits for investment in
government securities and investment in corporate debt.

36. Which of the following form part of Capital Budget of India?


1. Money earned by selling assets
2. Repayment of loans by states
3. Creating assets such as roads and hospitals
4. Loans to states

Select the correct answer using the code given below:

www.insightsactivelearn.com 35
Total Marks : 200
Test-25 (Subject)
( INSTA Prelims Test Series 2021 )

A. 1, 2 and 3 only
B. 2, 3 and 4 only
C. 1, 2 and 4 only
D. 1, 2, 3 and 4

Correct Answer : D

Answer Justification :

All the above statements are correct.

Revenue Budget– It consists of the Revenue Expenditure and Revenue Receipts.

Revenue Receipts are receipts which do not have a direct impact on the assets
and liabilities of the government. It consists of the money earned by the
government through tax (such as excise duty, income tax) and non-tax sources
(such as dividend income, profits, interest receipts).
Revenue Expenditure is the expenditure by the government which does not
impact its assets or liabilities. For example, this includes salaries, interest
payments, pension, and administrative expenses.

Capital Budget– It includes the Capital Receipts and Capital Expenditure.

Capital Receipts indicate the receipts which lead to a decrease in assets or an


increase in liabilities of the government. It consists of: (i) the money earned by
selling assets (or disinvestment) such as shares of public enterprises, and (ii) the
money received in the form of borrowings or repayment of loans by states.
Capital expenditure is used to create assets or to reduce liabilities. It
consists of: (i) the long-term investments by the government on creating assets
such as roads and hospitals, and (ii) the money given by the government in the
form of loans to states or repayment of its borrowings.

37. Consider the following statements regarding fiscal deficit under Union Budget 2021-22
1. The fiscal deficit is targeted at 9.5% of GDP in 2021-22.
2. The government’s aim to steadily reduce fiscal deficit to 4.5% of GDP by 2025-26.

Which of the statements given above is/are correct?


A. 1 only
B. 2 only
C. Both 1 and 2
D. Neither 1 nor 2

Correct Answer : B

Answer Justification :

www.insightsactivelearn.com 36
Total Marks : 200
Test-25 (Subject)
( INSTA Prelims Test Series 2021 )

In 2021-22, the government has not provided a target for the next three years and
will amend the FRBM Act to accommodate the higher fiscal deficit. The fiscal deficit
is targeted at 6.8% of GDP in 2021-22, down from the revised estimate of 9.5% in
2020-21 (4.6% in 2019-20). Hence, statement 1 is incorrect.

In the Union Budget 2021 speech, the Finance Minister has announced the
government’s aim to steadily reduce fiscal deficit to 4.5% of GDP by 2025-26.

The Fiscal Responsibility and Budget Management (FRBM) Bill was introduced in the
parliament of India in the year 2000 by Atal Bihari Vajpayee Government for providing legal
backing to the fiscal discipline to be institutionalized in the country. Subsequently, the FRBM
Act was passed in the year 2003. It is an act of the parliament that set targets for the
Government of India to establish financial discipline, improve the management of public funds,
strengthen fiscal prudence, and reduce its fiscal deficits.

38. Which of the following forms the major part of financing deficit in India?

A. Market borrowings and small savings


B. Market borrowings and external debt
C. Small savings and external debt
D. State provident funds and market borrowings

Correct Answer : A

Answer Justification :

www.insightsactivelearn.com 37
Total Marks : 200
Test-25 (Subject)
( INSTA Prelims Test Series 2021 )

Hence, option (a) is correct.

39. Consider the following statements regarding disinvestment under union budget 2021-22
1. Government has set target of Rs 1.75 lakh crore of disinvestment.
2. For the past three financial years, actual disinvestment proceeds have been more than the
budget estimates.

Which of the statements given above is/are correct?


A. 1 only
B. 2 only
C. Both 1 and 2
D. Neither 1 nor 2

Correct Answer : A

Answer Justification :

www.insightsactivelearn.com 38
Total Marks : 200
Test-25 (Subject)
( INSTA Prelims Test Series 2021 )

Hence, statement 2 is incorrect.

40. Consider the following statements regarding receipts and expenditure of Indian budget.
1. Direct and Indirect tax revenues together form less than 50% of total revenues.
2. Of the total expenditure, central sector schemes have the largest expenditure.

Which of the statements given above is/are correct?


A. 1 only
B. 2 only
C. Both 1 and 2
D. Neither 1 nor 2

Correct Answer : D

Answer Justification :

www.insightsactivelearn.com 39
Total Marks : 200
Test-25 (Subject)
( INSTA Prelims Test Series 2021 )

Hence, both statement 1 and 2 are incorrect.

41. Which of the following are the features of Fiscal Responsibility and Budget Management Act, 2003
(FRBMA)?
1. The FRBM rule set a target reduction of fiscal deficit to 3% of the GDP.
2. It mandated setting annual targets for the reduction of fiscal deficit and revenue deficit.
3. The RBI was supposed to subscribe to the primary issues of the central government securities
after 2006.

Which of the statements given above are correct?


A. 1 and 2 only
B. 2 and 3 only
C. 1 and 3 only
D. 1, 2 and 3

Correct Answer : A

www.insightsactivelearn.com 40
Total Marks : 200
Test-25 (Subject)
( INSTA Prelims Test Series 2021 )

Answer Justification :

The Fiscal Responsibility and Budget Management Act, 2003 (FRBMA) is an Act of the
Parliament of India to institutionalize financial discipline, reduce India’s fiscal deficit, improve
macroeconomic management and the overall management of the public funds by moving
towards a balanced budget.

Major Provisions of the FRBM Act, 2003

The FRBM rule set a target reduction of fiscal deficit to 3% of the GDP by
2008-09. This will be realized with an annual reduction target of 0.3% of GDP
per year by the Central government.
Revenue deficit has to be reduced by 0.5% of the GDP per year with complete
elimination by 2008-09.
Reduction of Public Debt
The government has to take appropriate measures to reduce the fiscal deficit
and revenue deficit so as to eliminate revenue deficit by 2008-09 and thereafter,
sizable revenue surplus has to be created.
It mandated setting annual targets for the reduction of fiscal deficit and
revenue deficit, contingent liabilities and total liabilities.
The government shall end its borrowing from the RBI except for temporary advances.
The RBI was supposed to not subscribe to the primary issues of the central
government securities after 2006.Hence, statement 3 is incorrect.
The revenue deficit and fiscal deficit may exceed the targets specified in the rules only
on grounds of national security, calamity and other exceptional grounds to be specified
by the Central government.
Amendments to FRBM Act: Fiscal Responsibility and Budget Management Act, 2003
was amended in 2012 that mandated the Central Government to lay before the Houses of
Parliament, Macro-Economic Framework Statement, Medium Term Fiscal Policy
Statement and Fiscal Policy Strategy Statement along with the Annual Financial
Statement and Demands for Grants.

42. Consider the following statements regarding Expenditure Management Commission


1. It is a statutory body under the Reserve Bank of India Act, 1934.
2. EMC will look into the expenditure reforms to be undertaken by the Government such as
review of the institutional arrangements including budgeting process.

Which of the statements given above is/are correct?


A. 1 only
B. 2 only
C. Both 1 and 2
D. Neither 1 nor 2

Correct Answer : B

www.insightsactivelearn.com 41
Total Marks : 200
Test-25 (Subject)
( INSTA Prelims Test Series 2021 )

Answer Justification :

The Government has constituted an Expenditure Management Commission (EMC)


through Resolution dated 4th September, 2014. The EMC has started its functioning.
Hence, statement 1 is incorrect.

EMC will look into various aspects of expenditure reforms to be undertaken by the
Government such as review of the institutional arrangements including budgeting
process and Fiscal Responsibility and Budget Management (FRBM) rules, suggest
ways to improve allocative efficiencies in the existing expenditure classification
system, and other issues concerning Public Expenditure Management.

43. Consider the following statements regarding foreign currency convertible bond (FCCB)
1. It is a type of bond that is issued in a currency other than the issuer's home currency.
2. A convertible bond is a mix between a debt and equity instrument.

Which of the statements given above is/are correct?


A. 1 only
B. 2 only
C. Both 1 and 2
D. Neither 1 nor 2

Correct Answer : C

Answer Justification :

All the above statements are correct.

What Is a Foreign Currency Convertible Bond?

A foreign currency convertible bond (FCCB) is a type of convertible bond issued in a currency
different than the issuer's domestic currency. In other words, the money being raised by the
issuing company is in the form of foreign currency. A convertible bond is a mix between a
debt and equity instrument. It acts like a bond by making regular coupon and principal
payments, but these bonds also give the bondholder the option to convert the bond into stock.

Key Takeaways

A foreign currency convertible bond (FCCB) is a type of bond that is issued in a


currency other than the issuer's home currency.
Convertible bonds fall in the middle of debt and equity financial instruments, both acting
as a bond but allowing investors to convert the bond into stock.
These kinds of bonds are often listed by large, multinational companies with offices
around the world, seeking to raise money in foreign currencies.

www.insightsactivelearn.com 42
Total Marks : 200
Test-25 (Subject)
( INSTA Prelims Test Series 2021 )

44. Consider the following statements regarding Institutional investment


1. Institutional investment is defined to be the investment done by institutions or organizations
such as banks, insurance companies, mutual fund houses in the financial or real assets of a
country.
2. Foreign institutional investors affect the net investment flows into the economy, while the
domestic institutional investors do not affect the net investment flows into the economy.

Which of the statements given above is/are correct?


A. 1 only
B. 2 only
C. Both 1 and 2
D. Neither 1 nor 2

Correct Answer : A

Answer Justification :

Definition: Domestic institutional investors are those institutional investors which undertake
investment in securities and other financial assets of the country they are based in.

Description: Institutional investment is defined to be the investment done by


institutions or organizations such as banks, insurance companies, mutual fund
houses, etc in the financial or real assets of a country. Simply stated, domestic
institutional investors use pooled funds to trade in securities and assets of their country.

These investment decisions are influenced by various domestic economic as well as political
trends. In addition to theforeign institutional investors, the domestic institutional
investors also affect the net investment flows into the economy. Hence, statement 2 is
incorrect.

45. Consider the following statements regarding Central Bank Digital currency (CBDC)
1. It is a legal tender and the Central bank’s liability in digital form appearing in its balance
sheet.
2. The Swedish Central Bank can be considered as the first Central Bank pioneered with the
introduction of CBDC in 2014.

Which of the statements given above is/are correct?


A. 1 only
B. 2 only
C. Both 1 and 2
D. Neither 1 nor 2

Correct Answer : A
www.insightsactivelearn.com 43
Total Marks : 200
Test-25 (Subject)
( INSTA Prelims Test Series 2021 )

Answer Justification :

Central Bank Digital currency (CBDC)is a legal tender (meaning recognized under
law as a means of payment/settlement) and the Central bank’s liability in digital form
appearing in its balance sheet. It has the backing of the Central Bank and the Sovereign.
CBDC is a form of electronic currency which can be exchanged at par with similarly
denominated cash and traditional deposit. It is another variant of the fiat money (currency)
which central banks traditionally issue. It carries the same promise like a traditional currency
note. For e.g. A INR 100 note bears a promise that this piece of paper is worth INR 100. The
RBI issuing a CBDC would be equivalent to converting the entire INR in the economy to
complete digital format backed and issued by the RBI being the monopoly supplier.

CBDC in other Economies:

The Bank of England can be considered as the first Central Bank pioneered with the
introduction of CBDC in 2014.Hence, statement 2 is incorrect.

However, China is likely to be the first economy to issue retail CBDC. China carried out a test
run of the Chinese CBDC by paying salaries for their government employees as CBDC in their
wallets. The Swedish Central Bank (Riksbank) is also looking forward to the issuance of the
digital version of Swedish Krona (e-Krona). Turkey has initiated R&D for having their digital
currency and aims to start pilots in the second half of 2021. In a recent Bank for International
Settlements (BIS) Paper, it was observed that 86% of the 65 respondent Central banks are
engaged in research or some kind of experimentation in CBDCs.

46. Consider the following statements regarding Gini Index


1. It is a measure of the distribution of income across a population.
2. The coefficient ranges from 0 to 1, with 1 representing perfect equality and 0 representing
perfect inequality.
3. The Gini index is often represented graphically through the Lorenz curve.

Which of the statements given above are correct?


A. 1 and 2 only
B. 2 and 3 only
C. 1 and 3 only
D. 1, 2 and 3

Correct Answer : C

Answer Justification :

What Is the Gini Index?

The Gini index, or Gini coefficient, is a measure of the distribution of income across a
population developed by the Italian statistician Corrado Gini in 1912. It is often used as

www.insightsactivelearn.com 44
Total Marks : 200
Test-25 (Subject)
( INSTA Prelims Test Series 2021 )

a gauge of economic inequality, measuring income distribution or, less commonly, wealth
distribution among a population. The coefficient ranges from 0 (or 0%) to 1 (or 100%),
with 0 representing perfect equality and 1 representing perfect inequality.Hence,
statement 2 is incorrect.

Values over 1 are theoretically possible due to negative income or wealth.

Key Takeaways

The Gini index is a measure of the distribution of income across a population.


A higher Gini index indicates greater inequality, with high income individuals receiving
much larger percentages of the total income of the population.
Global inequality as measured by the Gini index increased over the 19th and 20th
centuries, but has declined in more recent years.
Because of data and other limitations, the Gini index may overstate income inequality
and can obscure important information about income distribution.

Graphical Representation of the Gini Index

The Gini index is often represented graphically through the Lorenz curve, which shows
income (or wealth) distribution by plotting the population percentile by income on the
horizontal axis and cumulative income on the vertical axis. The Gini coefficient is equal to the
area below the line of perfect equality (0.5 by definition) minus the area below the Lorenz
curve, divided by the area below the line of perfect equality. In other words, it is double the
area between the Lorenz curve and the line of perfect equality.

47. Consider the following statements regarding Agriculture Census


1. It is a decadal census.
2. The basic unit of data collection in Agriculture Census is the operational holding.

Which of the statements given above is/are correct?


A. 1 only
B. 2 only
C. Both 1 and 2
D. Neither 1 nor 2

Correct Answer : B

Answer Justification :

Agriculture Census forms part of a broader system of collection of Agricultural Statistics in


India. It is a large-scale statistical operation for collection of requisite data and derivation of
quantitative information about the structural characteristics of agriculture in the country.
Through Agriculture Census, basic data on important aspects of agricultural economy
of operational holdings in the country is collected.

www.insightsactivelearn.com 45
Total Marks : 200
Test-25 (Subject)
( INSTA Prelims Test Series 2021 )

The basic unit of data collection in Agriculture Census is the operational holding.
Periodic Agriculture Census is important as it is the main source of information in the country
on basic characteristics of operational holdings such as land use, cropping pattern, irrigation
status and tenancy particulars. This information is tabulated by different size classes
(marginal, small, semi-medium, medium and large) and Social Groups including Scheduled
Castes/Scheduled Tribes, which are needed for development planning, socio-economic policy
formulation and establishment of national priorities. Census also provides the basis for
development of a comprehensive integrated national system of agricultural statistics and has
links with various components of national statistical system.

Hence, statement 1 is incorrect.

48. Which of the following has the highest weightage under Consumer Price Index urban?

A. Food and beverages


B. Clothing and footwear
C. Housing
D. Miscellaneous

Correct Answer : A

Answer Justification :

All India Consumer Price Indices

(Base: 2012=100)

Group Rural Urban Combined


Description
Code Weights Weights Weights

www.insightsactivelearn.com 46
Total Marks : 200
Test-25 (Subject)
( INSTA Prelims Test Series 2021 )

1 Food and beverages 54.18 36.29 45.86


2 Pan, tobacco and intoxicants 3.26 1.36 2.38
3 Clothing and footwear 7.36 5.57 6.53
4 Housing - 21.67 10.07
5 Fuel and light 7.94 5.58 6.84
6 Miscellaneous 27.26 29.53 28.32

Hence, option (a) is correct.

49. Consider the following statements regarding Index of Industrial Production (IIP)
1. The relative weights of these three sectors under IIP are 77.6% (manufacturing), 14.4%
(electricity) and 8% (mining).
2. The IIP data is compiled and published by CSO every month.

Which of the statements given above is/are correct?


A. 1 only
B. 2 only
C. Both 1 and 2
D. Neither 1 nor 2

Correct Answer : B

Answer Justification :

Index of Industrial Production (IIP)

Index of Industrial Production data or IIP as it is commonly called is an index that tracks
manufacturing activity in different sectors of an economy.
The IIP number measures the industrial production for the period under review, usually
a month, as against the reference period.
IIP is a key economic indicator of the manufacturing sector of the economy.
There is a lag of six weeks in the publication of the IIP index data after the reference
month ends.
IIP index is currently calculated using 2011-2012 as the base year.

IIP Index Components:

Mining, manufacturing, and electricity are the three broad sectors in which IIP
constituents fall.
The relative weights of these three sectors are 77.6% (manufacturing), 14.4%
(mining) and 8% (electricity).Hence, statement 1 is incorrect.
Electricity, crude oil, coal, cement, steel, refinery products, natural gas, and fertilizers
are the eight core industries that comprise about 40 per cent of the weight of items
included in the IIP.

www.insightsactivelearn.com 47
Total Marks : 200
Test-25 (Subject)
( INSTA Prelims Test Series 2021 )

Who releases IIP data?

The IIP data is compiled and published by CSO every month.


CSO or Central Statistical Organisation operates under the Ministry of Statistics and
Programme Implementation (MoSPI).

50. Considering money in circulation, which of the following is used as the most commonly used measure of
money supply?

A. M1
B. M2
C. M3
D. M4

Correct Answer : C

Answer Justification :

Legal Definitions: Narrow and Broad Money Money supply, like money demand, is a stock
variable. The total stock of money in circulation among the public at a particular point of time
is called money supply. RBI publishes figures for four alternative measures of money
supply, viz. M1, M2, M3 and M4. They are defined as follows

M1 = CU + DD

M2 = M1 + Savings deposits with Post Office savings banks

M3 = M1 + Net time deposits of commercial banks

M4 = M3 + Total deposits with Post Office savings organisations (excluding National Savings
Certificates)

where, CU is currency (notes plus coins) held by the public and DD is net demand deposits
held by commercial banks. The word ‘net’ implies that only deposits of the public held by the
banks are to be included in money supply. The interbank deposits, which a commercial bank
holds in other commercial banks, are not to be regarded as part of money supply. M1 and M2
are known as narrow money. M3 and M4 are known as broad money. These measures are in
decreasing order of liquidity. M1 is most liquid and easiest for transactions whereas M4 is
least liquid of all. M3 is the most commonly used measure of money supply. It is also
known as aggregate monetary resources.

Hence, option (c) is correct.

www.insightsactivelearn.com 48
Total Marks : 200
Test-25 (Subject)
( INSTA Prelims Test Series 2021 )

51. Which of the following forms the part of Capital account under international transactions?
1. Indian buys a UK Car Company
2. Sale of share of an Indian company to a Chinese customer
3. Receipt of loans from abroad

Which of the statements given above are correct?


A. 1 and 2 only
B. 2 and 3 only
C. 1 and 3 only
D. 1, 2 and 3

Correct Answer : D

Answer Justification :

ll the above statements are correct.

Capital Account

Capital Account records all international transactions of assets. An asset is any one of the
forms in which wealth can be held, for example: money, stocks, bonds, Government debt, etc.
Purchase of assets is a debit item on the capital account. If an Indian buy a UK Car Company,
it enters capital account transactions as a debit item (as foreign exchange is flowing out of
India). On the other hand, sale of assets like sale of share of an Indian company to a
Chinese customer is a credit item on the capital account. Fig. 6.2 classifies the items
which are a part of capital account transactions. These items are Foreign Direct Investments
(FDIs), Foreign Institutional Investments (FIIs), external borrowings and assistance.

Balance on Capital Account

Capital account is in balance when capital inflows (like receipt of loans from abroad, sale of
assets or shares in foreign companies) are equal to capital outflows (like repayment of loans,
purchase of assets or shares in foreign countries). Surplus in capital account arises when
capital inflows are greater than capital outflows, whereas deficit in capital account arises

www.insightsactivelearn.com 49
Total Marks : 200
Test-25 (Subject)
( INSTA Prelims Test Series 2021 )

when capital inflows are lesser than capital outflows.

52. Which of the following should be deducted from National Income to get Personal Income?
1. Undistributed profits
2. Net interest payments made by households
3. Transfer payments to the households from the government and firms
4. Corporate tax

Select the correct answer using the code given below:


A. 1, 2 and 3 only
B. 2, 3 and 4 only
C. 1, 2 and 4 only
D. 1, 3 and 4 only

Correct Answer : C

Answer Justification :

Personal Income is the part of National Income which is received by the households.

The formula for calculating Personal Income (PI) is

Personal income (PI) ≡ NI – Undistributed profits – Net interest payments made by


households – Corporate tax + Transfer payments to the households from the
government and firms.

Hence, statement 3 is incorrect.

Note

Undistributed profits – these are the profits which is not distributed to the
households

Corporate Tax – It also does not accrue to the households

53. Which of the following is the correct relation between Factor Price and Market Price?

A. Factor Price = Market Price + Indirect Taxes + Subsidies


B. Market Price = Factor Price - Indirect Taxes – Subsidies
C. Factor Price = Market Price + Indirect Taxes – Subsidies
D. Market Price = Factor Price + Indirect Taxes – Subsidies

www.insightsactivelearn.com 50
Total Marks : 200
Test-25 (Subject)
( INSTA Prelims Test Series 2021 )

Correct Answer : D

Answer Justification :

Factor Price and Market Price

Factor Price is total cost of all factors of production (such as labour, capital, land etc) used in
producing goods or services. It is the price of the commodity from the producers side.

When a commodity is produced, it is sold in the market

Market Price – It is the price at which a product is sold in the market. It includes the cost of
production in the form of wages, rent, interest, input prices, profit etc. It also includes the
taxes imposed by the government. It excludes Government subsidy.

Thus relationship between Factor price and Market price is

Market Price = Factor Price + Indirect Taxes – Subsidies

Hence, option (d) is correct.

54. Consider the following statements regarding Sarvodaya Plan


1. Sarvodaya Plan published in 1950 by Jayaprakash Narayan.
2. The plan drew its major inspirations from the Gandhian techniques of constructive works by
the community and trusteeship.

Which of the statements given above is/are correct?


A. 1 only
B. 2 only
C. Both 1 and 2
D. Neither 1 nor 2

Correct Answer : C

Answer Justification :

All the above statements are correct.

the Sarvodaya Plan

After the reports of the NPC were published and the government was set to go for the
five-year plans, a lone blueprint for the planned development of India was formulated
by the famous socialist leader Jayaprakash Narayan— the Sarvodaya Plan published in
January 1950. The plan drew its major inspirations from the Gandhian techniques of
constructive works by the community and trusteeship as well as the Sarvodaya concept

www.insightsactivelearn.com 51
Total Marks : 200
Test-25 (Subject)
( INSTA Prelims Test Series 2021 )

of Acharya Vinoba Bave, the eminent Gandhian constructive worker. Major ideas of the plan
were highly similar to the Gandhian Plan like emphasis on agriculture, agribased small and
cottage industries, self-reliance and almost no dependence on foreign capital and technology,
land reforms, self-dependent villages and decentralised participatory form of planning and
economic progress, to name the major ones. Some of the acceptable ideas of the plan got their
due importance when the Government of India promoted five year plans.

55. Consider the following statements regarding e-kuber


1. E-kuber is the core banking solution of the RBI that gives high degree of access to commercial
banks and other institutions to their current account with the RBI.
2. e-kuber enables banks access with their current account at any time, everywhere across the
country.
3. e-kuber is used to conduct exercises like auctioning of government securities.

Which of the statements given above is/are correct?


A. 1 only
B. 2 and 3 only
C. 3 only
D. 1, 2 and 3

Correct Answer : D

Answer Justification :

All the above statements are correct.

What is e-kuber?

The e-Kuber is the Core Banking Solution of the RBI which enables each bank to connect their
single current account across the country.

e-kuber

E-kuber is the core banking solution of the RBI that gives high degree of access
to commercial banks and other institutions to their current account with the
RBI.

e-kuber enables banks access with their current account at any time,
everywhere across the country.

The e-kuber is used by the RBI to execute various transactions with banks.

www.insightsactivelearn.com 52
Total Marks : 200
Test-25 (Subject)
( INSTA Prelims Test Series 2021 )

Utility of e-kuber is that it is used to conduct exercises like auctioning of


government securities.

Banks have current accounts with the RBI for facilitating their day to day functions. Banks can
get decentralised access to this account from anywhere-anytime using the e-kuber facility.

Core Banking Solutions (CBS) is a facility or arrangement that helps banks to offer large
number of customer-centric services on a 24×7 basis from a single location. The core banking
solution can support retail as well as corporate banking activities.

The e-kuber system was implemented in 2012, and it is considered as one of the successful
central banks administered Core Banking Systems in the world.

56. Consider the following statements regarding National Social Assistance Programme
1. It aims to the fulfill Article 41 and 42 of the Directive Principles of the Constitution.
2. National Social Assistance Programme was launched on 15th August, 1995.

Which of the statements given above is/are correct?


A. 1 only
B. 2 only
C. Both 1 and 2
D. Neither 1 nor 2

Correct Answer : C

Answer Justification :

All the above statements are correct.

National Social Assistance Programme

Introduction

NSAP stands for National Social Assistance Programme. NSAP was launched on 15th
August, 1995.

The National Social Assistance Programme (NSAP) represents a significant step


towards the fulfillment of the Directive Principles in Article 41 and 42 of the
Constitution recognizing the concurrent responsibility of the Central and the
State Governments in the matter. In particular, Article 41 of the Constitution of
India directs the State to provide public assistance to its citizens in case of

www.insightsactivelearn.com 53
Total Marks : 200
Test-25 (Subject)
( INSTA Prelims Test Series 2021 )

unemployment, old age, sickness and disablement and in other cases of


undeserved want within the limit of its economic capacity and development.

Objective of NSAP

National Social Assistance Programme is a social security and welfare programme to


provide support to aged persons, widows, disabled persons and bereaved families on
death of primary bread winner,
belonging to below poverty line households.

Components of NSAP

The NSAP at its inception in 1995 had three components namely

National Old Age Pension Scheme (NOAPS,

National Family Benefit Scheme (NFBS) and

National Maternity Benefit Scheme (NMBS). The National Maternity Benefit Scheme
(NMBS) was subsequently transferred on 1st April, 2001 from the Ministry of Rural
development to the Ministry of Health and Family Welfare.

On 1st April, 2000 a new Scheme known as Annapurna Scheme was launched. This scheme
aimed at providing food security to meet the requirement of those senior citizens who, though
eligible, have remained uncovered under the NOAPS.

In February 2009, two new Schemes known as Indira Gandhi National Widow Pension Scheme
(IGNWPS) and Indira Gandhi National Disability Pension Scheme (IGNDPS) were introduced.

Presently NSAP comprises of five schemes, namely –

Indira Gandhi National Old Age Pension Scheme (IGNOAPS),

Indira Gandhi National Widow Pension Scheme (IGNWPS),

Indira Gandhi National Disability Pension Scheme (IGNDPS),

National Family Benefit Scheme NFBS) and

www.insightsactivelearn.com 54
Total Marks : 200
Test-25 (Subject)
( INSTA Prelims Test Series 2021 )

Annapurna.

57. Which of the following are causes of Inflation?


1. Reduction in indirect taxes.
2. A mis-match between demand and supply pulls up prices.
3. High minimum wages to the employees.

Which of the statements given above is/are correct?


A. 1 only
B. 2 and 3 only
C. 3 only
D. 1, 2 and 3

Correct Answer : B

Answer Justification :

Why Inflation occurs

(a) Demand-Pull Inflation

A mis-match between demand and supply pulls up prices. Either the demand increases
over the same level of supply, or the supply decreases with the same level of demand and thus
the situation of demand-pull inflation arise. This was a Keynesian idea. The Keynesian School
suggests cuts in spending as the way of tackling excess demand mainly by increasing taxes
and reducing government expenditure.

In practice, the governments keep tracking the demand-supply matrix to check such inflation.
Depending upon the situation, the goods in short supply are imported, interest on loans
increased and wages revised.

(b) Cost-Push Inflation

An increase in factor input costs (i.e., wages and raw materials) pushes up prices. The price
rise which is the result of increase in the production cost is cost-push inflation. The Keynesian
school suggested controls on prices and incomes as direct ways of checking such an inflation
and, ‘moral suasions’ and measures to reduce the monopoly power of trade unions as indirect
measures (basically, cost-push inflations chiefly used to happen due to higher wage
demanded by the trade unions during the era). Today, the governments of the world
use many tools to check such inflations—reducing excise and custom duties on raw
materials, wage revisions, etc.

Hence, statement 1 is incorrect.

www.insightsactivelearn.com 55
Total Marks : 200
Test-25 (Subject)
( INSTA Prelims Test Series 2021 )

58. Consider the following statements regarding bottleneck inflation


1. This inflation takes place when the supply falls drastically and the demand remains at the
same level.
2. This situation arise due to supply-side hurdles, hazards or mismanagement.

Which of the statements given above is/are correct?


A. 1 only
B. 2 only
C. Both 1 and 2
D. Neither 1 nor 2

Correct Answer : C

Answer Justification :

All the above statements are correct.

bottleneck inflation

This inflation takes place when the supply falls drastically and the demand remains at
the same level. Such situations arise due to supply-side hurdles, hazards or
mismanagement which is also known as ‘structural inflation’. This could be put in the
‘demand-pull inflation’ category.

59. Which of the following are major economic traits of boom?


1. An accelerated and prolonged increase in the demand.
2. Demand peaks up to such a high level that it exceeds sustainable output/production levels.
3. The economy heats up and a demand supply lag is visible.

Which of the statements given above is/are correct?


A. 1 only
B. 2 and 3 only
C. 3 only
D. 1, 2 and 3

Correct Answer : D

Answer Justification :

All the above statements are correct.

Boom

www.insightsactivelearn.com 56
Total Marks : 200
Test-25 (Subject)
( INSTA Prelims Test Series 2021 )

A strong upward fluctuation in the economic activities is called boom.80 As economies try to
recover out of the phases of slowdown, recession and depression at times the measures taken
by the governments as well as the private sector might put economic activities as such which
the economic systems fail to digest. This is the phase of the boom. The major economic traits
of boom may be listed as given below:

(i) an accelerated and prolonged increase in the demand;

(ii) demand peaks up to such a high level that it exceeds sustainable


output/production levels;

(iii) the economy heats up and a demand supply lag is visible;

(iv) the market forces mismatch (i.e., demand and supply disequilibirium) and tend to
create a situation where inflation starts going upward;

(v) the economy might face structural problems like shortage of investible capital,
lower savings, falling standard of living, creation of a sellers’ market.

60. Consider the following statements regarding Non-Banking Financial Company (NBFC)
1. It should be a company registered under Banking Regulation Act, 1949.
2. It should have a minimum net owned fund of ₹ 200 crores.

Which of the statements given above is/are correct?


A. 1 only
B. 2 only
C. Both 1 and 2
D. Neither 1 nor 2

Correct Answer : D

Answer Justification :

What is a Non-Banking Financial Company (NBFC)?

A Non-Banking Financial Company (NBFC) is a company registered under the Companies Act,
1956 engaged in the business of loans and advances, acquisition of
shares/stocks/bonds/debentures/securities issued by Government or local authority or other
marketable securities of a like nature, leasing, hire-purchase, insurance business, chit
business but does not include any institution whose principal business is that of agriculture
activity, industrial activity, purchase or sale of any goods (other than securities) or providing
any services and sale/purchase/construction of immovable property. A non-banking institution
which is a company and has principal business of receiving deposits under any scheme or
arrangement in one lump sum or in installments by way of contributions or in any other
manner, is also a non-banking financial company (Residuary non-banking company).

www.insightsactivelearn.com 57
Total Marks : 200
Test-25 (Subject)
( INSTA Prelims Test Series 2021 )

NBFCs are doing functions similar to banks. What is difference between banks &
NBFCs?

NBFCs lend and make investments and hence their activities are akin to that of banks;
however there are a few differences as given below:

i. NBFC cannot accept demand deposits;

ii. NBFCs do not form part of the payment and settlement system and cannot issue cheques
drawn on itself;

iii. deposit insurance facility of Deposit Insurance and Credit Guarantee Corporation is not
available to depositors of NBFCs, unlike in case of banks.

What are the requirements for registration with RBI?

A company incorporated under the Companies Act, 1956 and desirous of commencing business
of non-banking financial institution as defined under Section 45 I(a) of the RBI Act, 1934
should comply with the following:

i. it should be a company registered under Section 3 of the companies Act, 1956

ii. It should have a minimum net owned fund of ₹ 200 lakh. (The minimum net owned
fund (NOF) required for specialized NBFCs like NBFC-MFIs, NBFC-Factors, CICs is
indicated separately in the FAQs on specialized NBFCs)

Hence, both statement 1 and 2 are incorrect.

61. Consider the following statements regarding Debenture Redemption Reserve


1. It is a provision stating that any Indian corporation that issues debentures must create a
debenture redemption service in an effort to protect investors from the possibility of a
company defaulting.
2. This is a provision under Securities and Exchange Board of India Act, 1992.

Which of the statements given above is/are correct?


A. 1 only
B. 2 only
C. Both 1 and 2
D. Neither 1 nor 2

Correct Answer : A

Answer Justification :

What is a Debenture Redemption Reserve?

www.insightsactivelearn.com 58
Total Marks : 200
Test-25 (Subject)
( INSTA Prelims Test Series 2021 )

A debenture redemption reserve (DRR) is a provision stating that any Indian


corporation that issues debentures must create a debenture redemption service in an
effort to protect investors from the possibility of a company defaulting. This provision
was tacked onto the Indian Companies Act of 1956, in an amendment introduced in
the year 2000. Hence, statement 2 is incorrect.

BREAKING DOWN Debenture Redemption Reserve

A debenture is a debt security that lets investors borrow money at a fixed interest rate. This
instrument is considered unsecured, because it is not backed by an asset, lien, or any other
form of collateral. Therefore, to protect debenture holders from the risk of default by the
issuing company, Section 117C of the Indian Companies Act of 1956 implemented the
debenture redemption reserve mandate. This capital reserve, which is to be funded by profits
issuers generate every year until the debentures are redeemed, must represent at least 25% of
the debenture's face value.

Key Takeaways

A debenture redemption reserve is requirement imposed on Indian corporation that


issue debentures, where they must create a debenture redemption service, to protect
investors from the possibility of a company defaulting.

This rule offers investors a measure of protection, because debentures are not backed by
an asset, a lien, or any other form of collateral.

The reserve must represent at least 25% of the face value of debentures issued.

62. Consider the following statements regarding Statutory Liquidity Ratio


1. SLR is a minimum percentage of deposits that a commercial bank has to maintain in the form
of liquid cash, gold or other securities.
2. The SLR was prescribed under the Banking Regulation Act, 1949.

Which of the statements given above is/are correct?


A. 1 only
B. 2 only
C. Both 1 and 2
D. Neither 1 nor 2

Correct Answer : C

Answer Justification :

All the above statements are correct.

www.insightsactivelearn.com 59
Total Marks : 200
Test-25 (Subject)
( INSTA Prelims Test Series 2021 )

Statutory Liquidity Ratio

Statutory Liquidity Ratio or SLR is a minimum percentage of deposits that a


commercial bank has to maintain in the form of liquid cash, gold or other securities.
It is basically the reserve requirement that banks are expected to keep before offering credit
to customers. These are not reserved with the Reserve Bank of India (RBI), but with
banks themselves. The SLR is fixed by the RBI. CRR (Cash Reserve Ratio) and SLR have
been the traditional tools of the central bank's monetary policy to control credit growth, flow
of liquidity and inflation in the economy. The SLR was prescribed by Section 24 (2A) of
Banking Regulation Act, 1949.

63. Consider the following statements regarding Market Stabilization scheme (MSS)
1. It is a monetary policy intervention by the RBI to withdraw excess liquidity by selling
government securities in the economy.
2. The MSS was introduced in 2014.

Which of the statements given above is/are correct?


A. 1 only
B. 2 only
C. Both 1 and 2
D. Neither 1 nor 2

Correct Answer : A

Answer Justification :

Market Stabilization scheme (MSS) is a monetary policy intervention by the RBI to


withdraw excess liquidity (or money supply) by selling government securities in the
economy. The MSS was introduced in April 2004. Hence, statement 2 is incorrect.

Main thing about MSS is that it is used to withdraw excess liquidity or money from the system
by selling government bonds.

Origin of MSS

Initially, the MSS was launched to withdraw the excess liquidity in the system that was
generated as a result of the RBI’s purchase of foreign currencies in the foreign exchange
market. From 2002 onwards, there was huge inflow of foreign capital into India. This led to
appreciation of rupee. Since appreciation is not good for exports, the RBI intervened in the
foreign exchange market by buying dollars. To buy dollars, the RBI has to give rupees. In this
way, high selling of rupees leads to excess liquidity (rupee) and thereby creating a potential
for inflation. To overcome this situation, the RBI has sold government bonds on a general basis
depending upon the volume of excess liquidity in the system. Here bonds go to financial
institutions and money goes back to the RBI. This withdrawal of excess liquidity is called
sterilisation.

www.insightsactivelearn.com 60
Total Marks : 200
Test-25 (Subject)
( INSTA Prelims Test Series 2021 )

During 2007-08, RBI sold Rs 2.5 lakh worth of securities in the financial system
implying that Rs 2.5 lakh crore money supply sterilised. Following facts are important to
understand MSS.

When MSS is to be used?

MSS is used when there is high liquidity in the system.

What securities to be sold under MSS?

The issued securities are government bonds and they are called as Market Stabilisation
Bonds (MSBs). Thus, the bonds issued under MSS are called MSBs. These securities are
owned by the government though they are issued by the RBI. It is to be remembered that
government is the owner of the securities. Usually, government’s securities (bonds/treasury
bills) are sold or issued by the RBI as the central bank is the banker to the government.

Whether the government can use the money obtained from MSS?

The money obtained under MSS should be kept with the RBI. It should not be
transferred to the government. This is because, if it is transferred, government will spend the
money in the economy thereby adding to liquidity.

64. Consider the following statements regarding Standing Deposit Facility


1. Government in the Budget’s (2021-22) Finance Act included a provision for the introduction of
the Standing Deposit Facility (SDF).
2. It allows the RBI to absorb liquidity (deposit) from commercial banks without giving
government securities in return to the banks.

Which of the statements given above is/are correct?


A. 1 only
B. 2 only
C. Both 1 and 2
D. Neither 1 nor 2

Correct Answer : B

Answer Justification :

The Standing Deposit Facility, proposed to be introduced by the RBI, is a collateral free
liquidity absorption mechanism that aims to absorb liquidity from the commercial banking
system into the RBI. Government in the Budget’s (2018) Finance Act included a
provision for the introduction of the Standing Deposit Facility (SDF). Hence,
statement 1 is incorrect.

What is Standing Deposit Facility (SDF)?

www.insightsactivelearn.com 61
Total Marks : 200
Test-25 (Subject)
( INSTA Prelims Test Series 2021 )

Standing Deposit Facility allows the RBI to absorb liquidity (deposit) from
commercial banks without giving government securities in return to the banks. In the
present situation, the main arrangement for the RBI to absorb excess money with the banking
system is the famous reverse repo mechanism. Under reverse repo (which is a part of the
Liquidity Adjustment Facility), banks will get government securities in return when they give
excess cash to the RBI. An interest rate of reverse repo rate is also provided to banks.

The inconvenience with this arrangement is that the RBI has to provide securities every time
when banks provide funds.

As per the stand of the RBI, when the central bank has to absorb tremendous amount of money
from the banking system through the reverse repo window, it will become difficult for the RBI
to provide such volume of government securities in return. This situation was occurred during
the time of demonetization.

In this sense, the Standing Deposit Facility (SDF) is a collateral free arrangement meaning
that RBI need not give collateral for liquidity absorption. The SDF will allow the RBI to suck
out liquidity without offering government securities as collateral.

65. Arrange the following chronologically based on the year of nationalization from the earliest to latest
1. State Bank of India
2. Punjab National Bank
3. Corporation Bank

Select the correct answer using the code given below:


A. 1 2 3
B. 3 1 2
C. 1 3 2
D. 2 1 3

Correct Answer : A

Answer Justification :

Before the steps of nationalization of Indian banks, only State Bank of India (SBI) was
nationalized. It took place in July 1955 under the SBI Act of 1955. Nationalization of Seven
State Banks of India (formed subsidiary) took place on 19th July, 1960.

Bank Names Year of Nationalization


Bank of Baroda 1969
Bank of India 1969
Bank of Maharashtra 1969
Canara Bank 1969
Central Bank of India 1969

www.insightsactivelearn.com 62
Total Marks : 200
Test-25 (Subject)
( INSTA Prelims Test Series 2021 )

Bank Names Year of Nationalization


Punjab National Bank 1969
Syndicate Bank 1969
Vijaya Bank 1969
Oriental Bank of Commerce 1980
Corporation Bank 1980

Hence, option (a) is correct.

66. Consider the following statements regarding Forward Markets Commission


1. It was a regulatory authority for commodity futures market in India, set up under Forward
Contracts (Regulation) Act 1952.
2. It was merged with SEBI.

Which of the statements given above is/are correct?


A. 1 only
B. 2 only
C. Both 1 and 2
D. Neither 1 nor 2

Correct Answer : C

Answer Justification :

All the above statements are correct.

Forward Markets Commission to merge with SEBI

In the first ever merger of two regulators, over 60-year-old FMC (Forward Markets
Commission) is all set to merge with the younger but much bigger capital markets watchdog
the Securities and Exchange Board of India (SEBI) to create a unified regulatory body.

Main aim: FMC’s merger with Sebi is aimed at streamlining the regulations and curb wild
speculations in the commodities market, while facilitating further growth there.

What made this merger necessary?

The commodities market has been known to be more prone to speculative activities
compared to the better-regulated stock market, while illegal activities like ‘dabba
trading’ have also been more frequent in this segment.

Besides, the high-profile NSEL scam has rocked this market in the recent past and the
subsequent regulatory and government interventions in this case eventually led to the

www.insightsactivelearn.com 63
Total Marks : 200
Test-25 (Subject)
( INSTA Prelims Test Series 2021 )

government announcing FMC’s merger with SEBI.

The merger was announced by Finance Minister Arun Jaitley in his budget speech.

Quick look:

SEBI was set up in 1988 as a non-statutory body for regulating the securities markets,
while it became an autonomous body in 1992 with fully independent powers.

FMC, on the other hand, has been regulating commodities markets since 1953, but lack
of powers has led to wild fluctuations and alleged irregularities remaining untamed in
this market segment.

At present, there are three national and six regional bourses for commodity futures in
the country.

This is the first major case of two regulators being merged, against the relatively more
frequent practice world wide of creating new regulatory authorities, including by carving out
new bodies from the existing entities.

About FMC:

Forward Markets Commission (FMC) is a regulatory authority for commodity futures market in
India. It is a statutory body set up under Forward Contracts (Regulation) Act 1952.

The Commission functions under the administrative control of the Ministry of Finance,
Department of Economic Affairs, Government of India.

Composition: The Act provides that the Commission shall consist of not less than two but not
exceeding four members appointed by the Central Government, out of them one being
nominated by the Central Government to be the Chairman of the Commission.

67. Which of the following are advantages of spot exchange?


1. Efficient price determination
2. Ensures transparency in price discovery
3. Bank finance will be available against the goods in the warehouse on easier terms.

Which of the statements given above are correct?


A. 1 and 2 only
B. 2 and 3 only
C. 1 and 3 only

www.insightsactivelearn.com 64
Total Marks : 200
Test-25 (Subject)
( INSTA Prelims Test Series 2021 )

D. 1, 2 and 3

Correct Answer : D

Answer Justification :

All the above statements are correct.

Advantages of spot exchange

Spot exchange provides various advantages over the traditional way of trading in commodities:

(i) Efficient price determination as price is determined by a wider cross-section of people from
across the country, unlike the traditional ‘mandis’ where price discovery for commodities used
to happen only through local participation.

(ii) Ensures transparency in price discovery— anonymity ensures convergence of different


price perceptions, as the buyer or seller merely expresses their desire to trade without even
meeting directly.

(iii) Ensures participation in large numbers by farmers, traders and processors across the
country and eliminate the possibility of cartelisation and other such unhealthy practices
prevalent in the commodity markets.

(iv) It brings in some best practices in commodity trading like, system of grading for quality,
creating network of warehouses with assaying facilities, facilitating trading in relatively
smaller quantities, lower transaction cost, etc.

(v) Bank finance available against the goods in the warehouse on easier terms improves
holding capacity and can actually incentivise farm production and hence reduce rural poverty.

(vi) Since the trades are guaranteed (by the exchange), counter party risk is avoided.

68. Who among the following are eligible under portfolio investment scheme (PIS)?
1. Non-Resident Indians (NRIs)
2. Persons of Indian Origin (PIOs)
3. Foreign Institutional Investors (FIIs)

Which of the statements given above is/are correct?


A. 1 only
B. 2 and 3 only
C. 3 only
D. 1, 2 and 3

Correct Answer : D

www.insightsactivelearn.com 65
Total Marks : 200
Test-25 (Subject)
( INSTA Prelims Test Series 2021 )

Answer Justification :

All the above statements are correct.

Foreign Institutional Investors (FIIs), Non-Resident Indians (NRIs), and Persons of


Indian Origin (PIOs) are allowed to invest in the primary and secondary capital
markets in India through the portfolio investment scheme (PIS). Under this scheme,
FIIs/NRIs can acquire shares/debentures of Indian companies through the stock exchanges in
India.

The ceiling for overall investment for FIIs is 24 per cent of the paid up capital of the Indian
company and 10 per cent for NRIs/PIOs. The limit is 20 per cent of the paid-up capital in the
case of public sector banks, including the State Bank of India.

The ceiling of 24 per cent for FII investment can be raised up to sectoral cap/statutory ceiling,
subject to the approval of the board and the general body of the company passing a special
resolution to that effect. And the ceiling of 10 per cent for NRIs/PIOs can be raised to 24 per
cent subject to the approval of the general body of the company passing a resolution to that
effect.

The ceiling for FIIs is independent of the ceiling of 10/24 per cent for NRIs/PIOs.

The equity shares and convertible debentures of the companies within the prescribed ceilings
are available for purchase under PIS subject to:

- the total purchase of all NRIs/PIOs both, on repatriation and non-repatriation basis, being
within an overall ceiling limit of (a) 24 per cent of the company's total paid up equity capital
and (b) 24 per cent of the total paid up value of each series of convertible debenture; and

- the investment made on repatriation basis by any single NRI/PIO in the equity shares and
convertible debentures not exceeding five per cent of the paid up equity capital of the
company or five per cent of the total paid up value of each series of convertible debentures
issued by the company.

https://www.rbi.org.in/fiilist/index.html

69. Consider the following statements regarding Credit Default Swap (CDS)
1. A credit default swap (CDS) is a financial derivative or contract that allows an investor to
"swap" or offset his or her credit risk with that of another investor.
2. They are customized between the two counterparties involved, which makes them opaque,
illiquid, and hard to track for regulators.

Which of the statements given above is/are correct?


A. 1 only
B. 2 only
C. Both 1 and 2
D. Neither 1 nor 2

www.insightsactivelearn.com 66
Total Marks : 200
Test-25 (Subject)
( INSTA Prelims Test Series 2021 )

Correct Answer : C

Answer Justification :

All the above statements are correct.

What is a Credit Default Swap (CDS)?

A credit default swap (CDS) is a financial derivative or contract that allows an


investor to "swap" or offset his or her credit risk with that of another investor. For
example, if a lender is worried that a borrower is going to default on a loan, the lender could
use a CDS to offset or swap that risk. To swap the risk of default, the lender buys a CDS from
another investor who agrees to reimburse the lender in the case the borrower defaults. Most
CDS contracts are maintained via an ongoing premium payment similar to the regular
premiums due on an insurance policy.

A credit default swap is the most common form of credit derivative and may involve municipal
bonds, emerging market bonds, mortgage-backed securities or corporate bonds.

Credit default swaps, or CDS, are credit derivative contracts that enable investors to
swap credit risk on a company, country, or other entity with another counterparty.

Credit default swaps are the most common type of OTC credit derivatives and are often
used to transfer credit exposure on fixed income products in order to hedge risk.

Credit default swaps are customized between the two counterparties involved,
which makes them opaque, illiquid, and hard to track for regulators

70. Consider the following statements regarding National Payments Corporation of India (NPCI)
1. It is an initiative of Reserve Bank of India (RBI) and Indian Banks’ Association (IBA)
2. It was created under Banking Regulation Act, 1949.

Which of the statements given above is/are correct?


A. 1 only
B. 2 only
C. Both 1 and 2
D. Neither 1 nor 2

Correct Answer : A

Answer Justification :

www.insightsactivelearn.com 67
Total Marks : 200
Test-25 (Subject)
( INSTA Prelims Test Series 2021 )

National Payments Corporation of India (NPCI), an umbrella organisation for


operating retail payments and settlement systems in India, is an initiative of Reserve
Bank of India (RBI) and Indian Banks’ Association (IBA) under the provisions of the
Payment and Settlement Systems Act, 2007, for creating a robust Payment &
Settlement Infrastructure in India. Hence, statement 2 is incorrect.

Considering the utility nature of the objects of NPCI, it has been incorporated as a “Not for
Profit” Company under the provisions of Section 25 of Companies Act 1956 (now Section 8 of
Companies Act 2013), with an intention to provide infrastructure to the entire Banking system
in India for physical as well as electronic payment and settlement systems. The Company is
focused on bringing innovations in the retail payment systems through the use of technology
for achieving greater efficiency in operations and widening the reach of payment systems.

The ten core promoter banks are State Bank of India, Punjab National Bank, Canara Bank,
Bank of Baroda, Union Bank of India, Bank of India, ICICI Bank, HDFC Bank, Citibank N. A.
and HSBC. In 2016 the shareholding was broad-based to 56 member banks to include more
banks representing all sectors.

71. Global Gender Gap Index 2021 was released by

A. UN Women
B. World Economic Forum
C. UNDP
D. World Bank

Correct Answer : B

Answer Justification :

Global Gender Gap Index 2021

This Index was released by the World Economic Forum since 2006.

The Gender Gap Index serves as a compass to track the progress on their towards
gender parity of the countries in four areas,

1. Political Empowerment,

2. Economic Participation and Opportunity,

3. Educational Attainment and

www.insightsactivelearn.com 68
Total Marks : 200
Test-25 (Subject)
( INSTA Prelims Test Series 2021 )

4. Health and Survival.

It also examines the drivers of gender gaps and outlines the policies and practices
needed for a gender-inclusive recovery.

The index scores the countries from 0 (inequality) to 1 (equality).

Hence, option (b) is correct.

72. Baikal-GVD (Gigaton Volume Detector) is a science project to detect

A. Aliens on Mars
B. Neutrino
C. Dark Matter
D. Solar Flares

Correct Answer : B

Answer Justification :

Hence, option (b) is correct.

Baikal-GVD (Gigaton Volume Detector):

Context:

Russian scientists recently launched one of the world’s biggest underwater neutrino
telescopes called the Baikal-GVD (Gigaton Volume Detector) in the waters of Lake
Baikail, the world’s deepest lake situated in Siberia.

About Baikal- GVD:

It is one of the three largest neutrino detectors in the world along with the
IceCube at the South Pole and ANTARES in the Mediterranean Sea.

It seeks to study in detail the elusive fundamental particles called neutrinos and
to possibly determine their sources.

What are neutrinos?

Neutrinos, first proposed by Swiss scientist Wolfgang Pauli in 1930, are the second most

www.insightsactivelearn.com 69
Total Marks : 200
Test-25 (Subject)
( INSTA Prelims Test Series 2021 )

widely occurring particle in the universe, only second to photons, the particle which
makes up light. In fact, neutrinos are so abundant among us that every second, there are more
than 100 trillion of them passing right through each of us — we never even notice them.

https://indianexpress.com/article/explained/explained-what-is-a-telescope-doing-inside-the-worl
ds-deepest-lake-7254805/lite/

73. Exercise SHANTIR OGROSHENA 2021 was a multinational military exercise held in

A. Turkey
B. Saudi Arabia
C. UAE
D. Bangladesh

Correct Answer : D

Answer Justification :

Exercise SHANTIR OGROSHENA 2021:

It is a multinational military exercise in Bangladesh.

Indian army will take part this year.

It will be held at Bangladesh to commemorate the birth centenary of Bangladesh ‘Father


of the Nation’ Bangabandhu Sheikh Mujibur Rahman.

Military observers from the USA, UK, Turkey, Kingdom of Saudi Arabia, Kuwait and
Singapore will also be in attendance throughout the exercise.

Hence, option (d) is correct.

74. Consider the following statements regarding Directorate of Film Festivals (DFF)
1. It was set up under the Ministry of Culture.
2. Dadasaheb Phalke Award is awarded under Directorate of Film Festivals.

Which of the statements given above is/are correct?


A. 1 only

www.insightsactivelearn.com 70
Total Marks : 200
Test-25 (Subject)
( INSTA Prelims Test Series 2021 )

B. 2 only
C. Both 1 and 2
D. Neither 1 nor 2

Correct Answer : B

Answer Justification :

The Directorate of Film Festivals (DFF) was set up under the Ministry of Information
& Broadcasting in 1973 with the objective of promotion of India films and cultural
exchange. Hence, statement 1 is incorrect.

DFF organizes and implements the following events and programmes to promote
Indian cinema :

i. The National Film Awards and the Dadasaheb Phalke Award.

ii. Organizing the annual International Film Festival of India (IFFI).

iii. Participation in Cultural Exchange Programmes and organizing screening of Indian


films through our Missions abroad.

iv. Selection of films for Indian Panorama.

v. Participation in International Film Festivals abroad.

vi. Organizing special film expositions such as retrospectives, Indian Panorama films
screenings and National Awards winning films screening and theme based film festivals in
different parts of the country.

vii. Collection, preservation and documentation of prints of Indian Panorama films for non-
commercial screenings.

https://dff.nic.in/AboutDFF.aspx

75. Consider the following statements regarding Emergency Credit Line Guarantee Scheme (ECLGS)
1. The scheme was launched as part of the Aatmanirbhar Bharat Abhiyan package.
2. 100% guarantee coverage is being provided by the National Credit Guarantee Trustee
Company.

Which of the statements given above is/are correct?


A. 1 only
B. 2 only
C. Both 1 and 2
D. Neither 1 nor 2

www.insightsactivelearn.com 71
Total Marks : 200
Test-25 (Subject)
( INSTA Prelims Test Series 2021 )

Correct Answer : C

Answer Justification :

All the above statements are correct.

Emergency Credit Line Guarantee Scheme (ECLGS):

Context:

The government has extended the ₹3-lakh-crore emergency credit line guarantee scheme
by another three months till June 30 and also widened its scope to new sectors, including
hospitality, travel and tourism.

Details:

ECLGS 3.0 would involve extending credit of up to 40% of total credit outstanding across
all lending institutions as on February 29, 2020.

The tenor of loans granted under ECLGS 3.0 would be 6 years, including a moratorium
period of 2 years.

About the scheme:

The scheme was launched as part of the Aatmanirbhar Bharat Abhiyan package
announced in May 2020 to mitigate the distress caused by coronavirus-induced
lockdown, by providing credit to different sectors, especially Micro, Small and
Medium Enterprises (MSMEs).

100% guarantee coverage is being provided by the National Credit Guarantee


Trustee Company, whereas Banks and Non-Banking Financial Companies (NBFCs)
provide loans.

The credit will be provided in the form of a Guaranteed Emergency Credit Line (GECL)
facility.

No Guarantee Fee shall be charged by NCGTC from the Member Lending Institutions
(MLIs) under the Scheme.

Interest rates under the Scheme shall be capped at 9.25% for banks and FIs, and at 14%
for NBFCs.

www.insightsactivelearn.com 72
Total Marks : 200
Test-25 (Subject)
( INSTA Prelims Test Series 2021 )

Eligibility:

Borrowers with credit outstanding up to Rs. 50 crore as on 29th February, 2020, and
with an annual turnover of up to Rs. 250 crore are eligible under the Scheme.

On 1st August 2020, the government widened the scope of the Rs. 3 lakh crore-ECLGS
scheme by doubling the upper ceiling of loans outstanding and including certain loans
given to professionals like doctors, lawyers and chartered accountants for business
purposes under its ambit.

https://epaper.thehindu.com/Home/MShareArticle?OrgId=GGQ8EDQEJ.1&imageview=0

76. Which of the following are the causes for vaccine wastage?
1. Not able to draw out the number of doses
2. Vials submerged in water
3. Suspected contamination
4. If it is exposed to heat

Select the correct answer using the code given below:


A. 1, 2 and 3 only
B. 2, 3 and 4 only
C. 1, 2 and 4 only
D. 1, 2, 3 and 4

Correct Answer : D

Answer Justification :

All the above statements are correct.

What is vaccine wastage, and how can it be prevented?

Context:

The Health Ministry has told the States and the Union Territories that there was no value in
conserving vaccines for the second dose and directed that prompt supply should be ensured to
all government and private hospitals where there was a demand.

What is Vaccine Wastage?

Vaccine wastage is an expected component of any large vaccination drive. But high vaccine
wastage inflates vaccine demand and increases unnecessary procurement.

www.insightsactivelearn.com 73
Total Marks : 200
Test-25 (Subject)
( INSTA Prelims Test Series 2021 )

Different stages where wastage occurs:

1. Cold chain points.

2. District vaccine stores.

3. Vaccination session site.

77. Consider the following statements regarding AIM-PRIME


1. AIM has joined hands with Bill & Melinda Gates Foundation (BMGF) to launch this nationwide
program.
2. It an initiative to promote and support science-based deep-tech start-ups & ventures across
India.

Which of the statements given above is/are correct?


A. 1 only

www.insightsactivelearn.com 74
Total Marks : 200
Test-25 (Subject)
( INSTA Prelims Test Series 2021 )

B. 2 only
C. Both 1 and 2
D. Neither 1 nor 2

Correct Answer : C

Answer Justification :

All the above statements are correct.

Atal Innovation Mission (AIM), NITI Aayog launched AIM-PRIME (Program for Researchers on
Innovations, Market-Readiness & Entrepreneurship), an initiative to promote and support
science-based deep-tech start-ups & ventures across India.

About:

In this regard, AIM has joined hands with Bill & Melinda Gates Foundation
(BMGF) to launch this nationwide program which will be implemented by
Venture Center - a non-profit technology business incubator.

The first cohort of the program is open to technology developers (early-stage deep tech
start-ups, and scientists/ engineers/ clinicians) with strong science-based deep tech
business ideas.

The program is also open to CEOs and Senior incubation managers of AIM Funded Atal
Incubation Centers that are supporting deep tech entrepreneurs. Deep technology is
an outcome of very intense research and development (R&D) with high knowledge
content.

The benefits of this program are aimed at addressing specific issues through training
and guidance over a period of 12 months.

https://pib.gov.in/PressReleasePage.aspx?PRID=1708798

78. Consider the following statements regarding New Development Bank (NDB)
1. It was established in 2014 at Fortaleza, Brazil.
2. Like the World Bank, NDB assigns votes based on capital share.

www.insightsactivelearn.com 75
Total Marks : 200
Test-25 (Subject)
( INSTA Prelims Test Series 2021 )

Which of the statements given above is/are correct?


A. 1 only
B. 2 only
C. Both 1 and 2
D. Neither 1 nor 2

Correct Answer : A

Answer Justification :

New Development Bank (NDB):

Context:

Finance and Corporate Affairs Minister Nirmala Sitharaman has urged the New
Development Bank (NDB) to consider working closely with India’s new development
financing institution for funding infrastructure.

Background:

NDB has so far approved 18 projects in India, including emergency loans of $2 billion to
support health spending and economic recovery in the aftermath of the COVID-19 pandemic.

About NDB:

It is a multilateral development bank operated by the BRICS states (Brazil, Russia, India,
China and South Africa).

It was agreed to by BRICS leaders at the 5th BRICS summit held in Durban, South
Africa in 2013.

It was established in 2014, at the 6th BRICS Summit at Fortaleza, Brazil.

The bank is set up to foster greater financial and development cooperation among the
five emerging markets.

Headquartered in Shanghai, China.

In 2018, the NDB received observer status in the United Nations General Assembly,
establishing a firm basis for active and fruitful cooperation with the UN.

Voting:

Unlike the World Bank, which assigns votes based on capital share, in the New

www.insightsactivelearn.com 76
Total Marks : 200
Test-25 (Subject)
( INSTA Prelims Test Series 2021 )

Development Bank each participant country will be assigned one vote, and none of
the countries will have veto power. Hence, statement 2 is incorrect.

Roles and functions:

The Bank will mobilize resources for infrastructure and sustainable development projects in
BRICS and other emerging economies and developing countries, to supplement existing efforts
of multilateral and regional financial institutions for global growth and development.

https://epaper.thehindu.com/Home/ShareArticle?OrgId=GHO8E9ODB.1&imageview=0

79. Human Rights Report was released by

A. U.S. State Department


B. Amnesty International
C. United Nations Human Rights Council
D. Economist Intelligence Unit

Correct Answer : A

Answer Justification :

2020 Human Rights Report:

Context:

Released by the U.S. State Department.

The report, which is submitted each year to the U.S. Congress, is retrospective and
contains a country-wise discussion of the state of human rights.

Key findings:

1. The harassment and detention of journalists critical of the Indian government in their
reporting and on social media, has continued, although the government generally
respected the freedom of expression.

2. Government’s requests for user data from Internet companies had increased
“dramatically.”

3. There were several instances in which the government, or actors considered close to the
government, allegedly pressured or harassed media outlets critical of the government,

www.insightsactivelearn.com 77
Total Marks : 200
Test-25 (Subject)
( INSTA Prelims Test Series 2021 )

including through online trolling.

4. The government made 49,382 user data requests in 2019 from Facebook, a 32% increase
from 2018. Over the same period, Google requests increased by 69%, while Twitter
requests saw a 68% increase.

https://epaper.thehindu.com/Home/ShareArticle?OrgId=GDO8E9S7V.1&imageview=0

Hence, option (a) is correct.

80. Consider the following statements regarding Sabarmati River


1. It originates from Aravalli hills in Udaipur District of state Rajasthan.
2. Ahmedabad city is located along the bank of this river.

Which of the statements given above is/are correct?


A. 1 only
B. 2 only
C. Both 1 and 2
D. Neither 1 nor 2

Correct Answer : C

Answer Justification :

All the above statements are correct.

Sabarmati River Front Development project:

It is an environmental improvement, social uplift and urban rejuvenation project that will
renew Ahmedabad. The project will reclaim approximately 200 hectares of land from the
riverbed.

About the Sabarmati river:

It is one of the west flowing rivers along with Narmada and Tapti.

It Originates from Aravalli hills near Tepur village in Udaipur Dist of state Rajasthan.

Mouth of the river: Gulf of Cambey ( Khambhat).

Ahmedabad city is located along the bank of this river.

www.insightsactivelearn.com 78
Total Marks : 200
Test-25 (Subject)
( INSTA Prelims Test Series 2021 )

https://indianexpress.com/article/explained/explained-what-is-the-sabarmati-river-front-develop
ment-project-what-it-seeks-to-achieve-7249497/

81. Ānandam: The Center for Happiness was inaugurated in the state/UT of

A. Jammu and Kashmir


B. Sikkim
C. Telangana
D. Puducherry

Correct Answer : A

Answer Justification :

Union Education Minister inaugurates Ānandam: The Center for Happiness in IIM Jammu.
Hence, option (a) is correct.

Incorporating happiness in academic curriculum crucial towards empowering our nation- Shri
Ramesh Pokhriyal ‘Nishank’

The Center for Happiness aligns with the National Education Policy 2020, aims to achieve
holistic well-being for all - Union Education Minister.

Speaking on the occasion Shri Pokhriyal congratulated IIM Jammu for the new venture and
defined the need for Ānandam: The Center for Happiness. He said that incorporating
happiness in the academic curriculum of students is a very crucial step towards empowering
our nation. This step will take our education system to newer heights, similar to the times of
ancient Indian universities like Nalanda and Takshashila. He explained that how Anandam:
The Center for happiness aligns with the National Education Policy 2020 that aims to
transform India’s education system by 2021. He concluded his speech by encouraging other
institutions in our country to have their own center for happiness, to help students live a
stress-free life.

The Center for Happiness acquires its name ‘Ānandam’ from the Indian wisdom tradition
where the pure consciousness is being as “anandam”. “Ānandam” doesn’t aim for only
happiness but knowing the truth, doing good and enjoying the beauty around. The tagline for
“Ānandam” further reinforces its ideology that it will work for the well-being of all. The tagline
“Sarvabhūtahiteratāḥ” stands for being always engaged for the well-being of all.

https://pib.gov.in/PressReleasePage.aspx?PRID=1708428

www.insightsactivelearn.com 79
Total Marks : 200
Test-25 (Subject)
( INSTA Prelims Test Series 2021 )

82. Greater Tipraland is a demand made by certain section of

A. Tripura
B. Nagaland
C. Meghalaya
D. Mizoram

Correct Answer : A

Answer Justification :

What is the Greater Tipraland demand?

Context:

Demand for ‘Greater Tipraland’ has been the main issue in the campaigns for the elections to
the Tripura Tribal Areas Autonomous District Council. Hence, option (a) is correct.

www.insightsactivelearn.com 80
Total Marks : 200
Test-25 (Subject)
( INSTA Prelims Test Series 2021 )

What is Greater Tipraland?

‘Greater Tipraland’ is essentially an extension of the ruling tribal partner Indigenous Peoples
Front of Tripura – IPFT’s demand of Tipraland, which sought a separate state for tribals of
Tripura.

The new demand seeks to include every tribal person living in indigenous area or village
outside the Tripura Tribal Areas Autonomous District Council (TTAADC) under the
proposed model.

However, the idea doesn’t restrict to simply the Tripura tribal council areas, but seeks to
include ‘Tiprasa’ of Tripuris spread across different states of India like Assam, Mizoram
etc. as well, even those living in Bandarban, Chittagong, Khagrachari and other
bordering areas of neighbouring Bangladesh.

www.insightsactivelearn.com 81
Total Marks : 200
Test-25 (Subject)
( INSTA Prelims Test Series 2021 )

https://epaper.thehindu.com/Home/ShareArticle?OrgId=GSU8E3HTC.1&imageview=0

83. Consider the following statements regarding electoral bonds


1. Electoral bonds are interest-free bearer instruments used to donate money anonymously to
political parties.
2. The bonds are sold in multiples of Rs 1,000, Rs 10,000, Rs 1 lakh, Rs 10 lakh and Rs 1 crore.
3. State Bank of India is the only bank authorized to sell them.

Which of the statements given above are correct?


A. 1 and 2 only
B. 2 and 3 only
C. 1 and 3 only
D. 1, 2 and 3

Correct Answer : D

Answer Justification :

All the above statements are correct.

What are electoral bonds?

Announced in the 2017 Union Budget, electoral bonds are interest-free bearer
instruments used to donate money anonymously to political parties.

A bearer instrument does not carry any information about the buyer or payee.

The holder of the instrument (which is the political party) is presumed to be its
owner.

The bonds are sold in multiples of Rs 1,000, Rs 10,000, Rs 1 lakh, Rs 10 lakh, and Rs 1
crore, and State Bank of India is the only bank authorised to sell them.

Donors can buy and subsequently donate bonds to a political party, which can encash
the bonds through its verified account within 15 days.

There is no limit on the number of bonds an individual or company can purchase.

If a party hasn’t enchased any bonds within 15 days, SBI deposits these into the Prime
Minister’s Relief Fund.

www.insightsactivelearn.com 82
Total Marks : 200
Test-25 (Subject)
( INSTA Prelims Test Series 2021 )

What is the EC’s stand?

The EC had objected to amendments in the Representation of the People Act that
exempt political parties from disclosing donations through this route.

In a situation where the contribution received through electoral bonds are not reported,
on perusal of the contribution report of political parties, it cannot be ascertained
whether the political party has taken any donation in violation of provision under
Section 29(b) of the RP Act which prohibits the political parties from taking donations
from government companies and foreign sources.

https://indianexpress.com/article/explained/how-the-electoral-bonds-scheme-has-worked-so-far-
and-why-it-has-been-challenged-in-sc-7247184/

84. Consider the following statements regarding Development of Iconic Tourist Destinations Scheme
1. It is a centrally sponsored scheme for the development of identified iconic destinations in the
country following a holistic approach.
2. It was introduced by Ministry of Tourism.

Which of the statements given above is/are correct?


A. 1 only
B. 2 only
C. Both 1 and 2
D. Neither 1 nor 2

Correct Answer : B

Answer Justification :

Development of Iconic Tourist Destinations Scheme:

Context:

Tourism Ministry, in association with Madhya Pradesh Tourism and India Convention
Promotion Bureau are organizing ‘MICE (MICE – Meetings, Incentives, Conferences and
Exhibitions) Roadshow – Meet in India’ at Khajuraho, Madhya Pradesh.

About the Scheme:

It is a central sector scheme for the development of identified iconic


destinations in the country following a holistic approach.

www.insightsactivelearn.com 83
Total Marks : 200
Test-25 (Subject)
( INSTA Prelims Test Series 2021 )

The objective of the scheme is to boost the tourism influx in India and serve as a model
for other tourism sites.

The nodal agency for the scheme is the Tourism Ministry while other ministries such as
civil aviation, railways, etc. are also involved.

The Ministry of Tourism has framed ‘Development of Iconic Tourist Destinations


Scheme’ a Central Sector Scheme for development of nineteen identified iconic
destinations in the country following a holistic approach. Hence, statement 1 is
incorrect.

The identified tourist sites to be developed as iconic destinations under the scheme are Taj
Mahal & Fatehpur Sikri (Uttar Pradesh), Ajanta Caves & Ellora Caves (Maharashtra),
Humayun’s Tomb, Red Fort & Qutub Minar (Delhi), Colva Beach (Goa), Amer Fort (Rajasthan),
Somnath, Dholavira & Statue of Unity (Gujarat), Khajuraho (MP), Hampi (Karnataka),
Mahabalipuram (TamilNadu), Kaziranga (Assam), Kumarakom (Kerala), Konark (Odisha) and
Mahabodhi Temple (Bihar).

https://pib.gov.in/PressReleaseIframePage.aspx?PRID=1707436

85. Startsapuk Tso and Tso Kar lake complex are located in the state/UT of

A. Ladakh
B. Uttarakhand
C. Sikkim
D. Arunachal Pradesh

Correct Answer : A

Answer Justification :

Ladakh's Startsapuk Tso, Tso Kar lake complex added to list of Ramsar site

The latest site to be added to the list is a high-altitude wetland complex of two connected
lakes, Startsapuk Tso and Tso Kar, in Ladakh. Hence, option (a) is correct.

One more wetland in India has been added to the list of recognised sites of international
importance under the treaty of Ramsar Convention, taking the number of such wetlands in the
country to 42, the highest in South Asia.

The latest site to be added to the list is a high-altitude wetland complex of two connected

www.insightsactivelearn.com 84
Total Marks : 200
Test-25 (Subject)
( INSTA Prelims Test Series 2021 )

lakes, Startsapuk Tso and Tso Kar, in Ladakh.

In a tweet, Union Environment Prakash Javadekar said, "Happy to share that high-altitude
wetland complex in Changthang region of Ladakh is recognised as wetland of international
importance. The complex is a notable example of two connected lakes, the freshwater
Startsapuk Tso and the hypersaline Tso Kar. Now, India has 42 Ramsar sites."

Before that, Kabartal in Bihar's Begusarai district was recognised as a wetland of international
importance under the Ramsar Convention. It was the first such site in the state to figure in the
list, according to the Union Environment Ministry.

https://www.business-standard.com/article/pti-stories/ladakh-s-startsapuk-tso-tso-kar-lake-com
plex-added-to-list-of-ramsar-site-120122400524_1.html

86. Consider the following statements regarding Swachhata Abhiyan Mobile Application
1. It was launched by Ministry of Jal Shakti.
2. This mobile application has been developed to identify and geotag the data of insanitary
latrines.

Which of the statements given above is/are correct?


A. 1 only
B. 2 only
C. Both 1 and 2
D. Neither 1 nor 2

Correct Answer : B

Answer Justification :

Union Minister for Social Justice and Empowerment Shri Thaawarchand Gehlot
launched a Mobile Application “Swachhata Abhiyan” in the presence of Ministers of
State for Social Justice and Empowerment Shri Ram Das Athawale and Shri Krishan
Pal Gurjar in New Delhi today. In the absence of any authentic data base regarding
the location of insanitary latrines, it has been decided to seek the help of NGOs,
Social Organisations and general public for collection and compilation of the data. A
mobile application “Swachhata Abhiyan” has been developed for this purpose. This
mobile application can be downloaded from the ‘Google Play Store’ free of cost.
Hence, statement 1 is incorrect.

Addressing on the occasion, Shri Thaawarchand Gehlot said that this mobile application

www.insightsactivelearn.com 85
Total Marks : 200
Test-25 (Subject)
( INSTA Prelims Test Series 2021 )

“Swachhata Abhiyan” has been developed to identify and geotag the data of insanitary latrines
and manual scavengers so that the insanitary latrines can be replaced with sanitary latrines
and rehabilitate all the manual scavengers to provide dignity of life to them. The mobile
application can be downloaded from the ‘Google Play Store’ on your mobiles. He urged all
citizens of the country to download the above application and if they notice any insanitary
latrine or manual scavenger, their details may be uploaded using the application.

https://pib.gov.in/PressReleasePage.aspx?PRID=1683291

87. DigiBoxx was initiated by

A. Microsoft
B. Google
C. NITI aayog
D. RBI

Correct Answer : C

Answer Justification :

India now has its own cloud storage platform, DigiBoxx. A potential alternative to Google
Drive, Dropbox, and Microsoft OneDrive, the indigenous cloud service is seen as a step that
furthers the initiative of ‘Atmanirbhar Bharat.’ The homegrown data storage and
management platform has been launched by NITI Aayog and it will provide affordable
services to both regular users as well as businesses. Without revealing any technical
details, the company said that all the data will be encrypted and stored in servers in
India. Hence, option (c) is correct.

The cloud storage platform is available on Android and iOS and can be accessed on computers
through the web. The most interesting feature of the service DigiBoxx is that it allows users to
share files even when they don’t have a DigiBoxx account. The feature has been named
InstaShare and allows users to share up to 2GB of content for free. All you need to have is your
mobile number or email-ID. Once you upload your file, a link will be generated that can be
used by the recipient to download the content. The data remains available on the link for 45
days post which it will be deleted and the link becomes invalid, the company said.

https://www.financialexpress.com/industry/technology/niti-aayog-launches-digiboxx-indias-ans
wer-to-dropbox-and-google-drive/2155893/

88. Consider the following statements regarding Amended Technology Upgradation Fund Scheme (ATUFS)
1. It is a credit linked Capital Investment Subsidy (CIS) scheme.
2. It was launched to catalyze capital investments for technology upgradation and modernization
of the food processing industry.

www.insightsactivelearn.com 86
Total Marks : 200
Test-25 (Subject)
( INSTA Prelims Test Series 2021 )

Which of the statements given above is/are correct?


A. 1 only
B. 2 only
C. Both 1 and 2
D. Neither 1 nor 2

Correct Answer : A

Answer Justification :

The Government is implementing Amended Technology Upgradation Fund Scheme


(ATUFS), a credit linked Capital Investment Subsidy (CIS) scheme during 2016 to
2022 with an outlay of Rs. 17822 crore to catalyze capital investments for technology
upgradation and modernization of the textile industry. Hence, statement 2 is
incorrect.

The scheme promotes ease of doing business in the country and achieve the vision of
generating employment and promoting exports through “Make in India’’ with "Zero effect and
Zero defect" in manufacturing. The scheme facilitates augmenting of investment, productivity,
quality, employment, exports along with import substitution in the textile industry. It also
indirectly promotes investment in textile machinery (having benchmarked technology)
manufacturing.

Ministry has also notified the Scheme for Production and Employment Linked Support for
Garmenting Units (SPELSGU) under ATUFS to incentivize production and employment
generation in the garmenting sector vide Resolution dated 25.07.2016.

https://pib.gov.in/PressReleseDetail.aspx?PRID=1606086

89. Inner Line permit is applicable in which of the following states?


1. Arunachal Pradesh
2. Nagaland
3. Mizoram

Which of the statements given above are correct?


A. 1 and 2 only
B. 2 and 3 only
C. 1 and 3 only
D. 1, 2 and 3

Correct Answer : D

Answer Justification :

www.insightsactivelearn.com 87
Total Marks : 200
Test-25 (Subject)
( INSTA Prelims Test Series 2021 )

All the above statements are correct.

Arunachal Pradesh, Nagaland and Mizoram are protected by the Inner Line, and
lately Manipur was added. The concept originates from the Bengal Eastern Frontier
Regulation Act (BEFR), 1873.

What is Inner Line?


A concept drawn by colonial rulers, the Inner Line separated the tribal-populated hill areas in
the Northeast from the plains. To enter and stay for any period in these areas, Indian citizens
from other areas need an Inner Line Permit (ILP). Arunachal Pradesh, Nagaland and Mizoram
are protected by the Inner Line, and lately Manipur was added. The concept originates from
the Bengal Eastern Frontier Regulation Act (BEFR), 1873.

“The policy of exclusion first came about as a response to the reckless expansion of British
entrepreneurs into new lands which threatened British political relations with the hill tribes,”
wrote the political scientist Sanjib Baruah in his book India Against Itself: Assam and the
Politics of Nationality. The BEFR prohibits an outsider’s — “British subject or foreign citizen”
— entry into the are beyond the Inner Line without a pass and his purchase of land there. On
the other hand, the Inner Line also protects the commercial interests of the British from the
tribal communities.

After Independence, the Indian government replaced “British subjects” with “Citizen of India”.
In 2013, the Home Ministry told Rajya Saha, “The main aim of ILP system is to prevent
settlement of other Indian nationals in the States where ILP regime is prevalent, in order to
protect the indigenous/tribal population.”

https://indianexpress.com/article/explained/inner-line-permit-what-is-its-caa-context-6441390/

90. Consider the following statements regarding E-Sampada


1. It was launched by Ministry of Housing and Urban Affairs.
2. It provides a single window for allotment for over one lakh government residential
accommodations.

Which of the statements given above is/are correct?


A. 1 only
B. 2 only
C. Both 1 and 2
D. Neither 1 nor 2

Correct Answer : C

www.insightsactivelearn.com 88
Total Marks : 200
Test-25 (Subject)
( INSTA Prelims Test Series 2021 )

Answer Justification :

All the above statements are correct.

Ministry of Housing & Urban Affairs

Booking of Holiday Homes, Social Function Venues Like 5 Ashoka Road, Allotment for over
One Lakh Government Residential Accommodations Integrated on New Platform

E-Sampada- New Web Portal & Mobile App Launched on Good Governance Day-- To Simplify
Processes & Bring Uniformity In Systems Across India

Four Websites & Two Mobile Apps of Directorate of Estates Integrated into one--all services on
same Platform throughout Country

E-Governance to Boost Transparency & Accountability in Providing Various Estate Services

Good governance day is celebrated annually on 25 December, the anniversary of the birth of
former Prime Minister Atal Bihari Vajpayee.

In line with the objectives of boosting transparency and accountability while ensuring ease of
living for the citizens, the Directorate of Estates, Ministry of Housing and Urban Affairs today
dedicated to the nation, a new Web Portal and Mobile App, e-Sampada. The new application
provides a single window for all these services including allotment for over one lakh
government residential accommodations, office space allotment to government
organisations in 45 office complexes in 28 cities, booking of 1,176 holiday home
rooms and venues like 5, Ashoka Road for social functions etc.

https://pib.gov.in/PressReleaseIframePage.aspx?PRID=1683601

91. Monpa Handmade Paper Industry is a heritage art form of

A. Arunachal Pradesh
B. Chhattisgarh
C. Orisha
D. Jharkhand

Correct Answer : A

Answer Justification :

KVIC Brings Alive 1000-yrs Old Monpa Handmade Paper Industry in Tawang to
Revive the Heritage Art; a Historic Feat for North East. Hence, option (a) is correct.

The 1000-year old heritage art – the Monpa Handmade Paper of Arunachal Pradesh – which

www.insightsactivelearn.com 89
Total Marks : 200
Test-25 (Subject)
( INSTA Prelims Test Series 2021 )

was driven to the extinction, has come to life once again, with the committed efforts of Khadi
and Village Industries Commission (KVIC).

The art of making Monpa handmade paper originated over 1000 years ago. Gradually the art
became an integral part of local custom and culture in Tawang in Arunachal Pradesh. Once
produced in every household in Tawang, this handmade paper was a major source of livelihood
for the locals. However, the handmade paper industry almost disappeared in the last 100
years; prompting KVIC to plan revival of this ancient art.

https://pib.gov.in/PressReleaseIframePage.aspx?PRID=1683797

92. Consider the following statements regarding Joint Comprehensive Plan of Action (JCPOA)
1. It was endorsed by UN Security Council.
2. Iran’s compliance with the nuclear-related provisions of the JCPOA is verified by the
International Atomic Energy Agency (IAEA).

Which of the statements given above is/are correct?


A. 1 only
B. 2 only
C. Both 1 and 2
D. Neither 1 nor 2

Correct Answer : C

Answer Justification :

All the above statements are correct.

The Joint Comprehensive Plan of Action (JCPOA) is a detailed, 159-page agreement


with five annexes reached by Iran and the P5+1 (China France, Germany, Russia, the
United Kingdom, and the United States) on July 14, 2015. The nuclear deal was
endorsed by UN Security Council Resolution 2231, adopted on July 20, 2015. Iran’s
compliance with the nuclear-related provisions of the JCPOA is verified by the
International Atomic Energy Agency (IAEA) according to certain requirements set forth in
the agreement.

Timeline for Implementation

July 14, 2015, Finalization Day: conclusion of the agreement. Finalization day triggers
Iran and the United States to begin domestic review processes of the JCPOA. Iran also
begins providing the IAEA with information necessary for the agency to complete its
investigation into past activities related to nuclear weapons development.

www.insightsactivelearn.com 90
Total Marks : 200
Test-25 (Subject)
( INSTA Prelims Test Series 2021 )

October 18, 2015, Adoption Day: 90 days after the passage of the UN Security
Council Resolution endorsing the deal (July 20, 2015). Adoption day triggers Iran and the
P5+1 to take steps (outlined below) to meet the commitments to fully implement the
JCPOA.

January 16, 2016, Implementation Day: the IAEA certifies that Iran has taken the key
steps to restrict its nuclear program and has put in place increased monitoring. The
IAEA's report on implementation day triggers U.S., EU, and UN sanctions relief.

October 2023, Transition Day: Eight years after adoption day (or the IAEA reaching
its broader conclusion on Iran's nuclear program, whichever is sooner). Transition day
triggers the UN to lift missile restrictions, Iran to seek ratification of its additional
protocol, the EU to terminate all remaining nuclear sanctions, United States to remove
certain entities from the sanctioned list, and the United States to seek legislative
termination of certain sanctions.

October 2025, Termination Day: Ten years after adoption day. Termination day
terminates Resolution 2231 and the Security Council closes Iran's nuclear file.

https://www.armscontrol.org/factsheets/JCPOA-at-a-glance

93. Consider the following statements regarding Indian pangolin


1. It has been listed in Schedule I of Wildlife (Protection) Act, 1972.
2. It has been listed as Critically Endangered on IUCN Red List.

Which of the statements given above is/are correct?


A. 1 only
B. 2 only
C. Both 1 and 2
D. Neither 1 nor 2

Correct Answer : A

Answer Justification :

www.insightsactivelearn.com 91
Total Marks : 200
Test-25 (Subject)
( INSTA Prelims Test Series 2021 )

Indian pangolin
Key Facts

Common Name

Indian pangolin

Scientific Name

Manis crassicaudata

Length

Head and body : 60-70 cm

Weight

9- 18 kg

Population

Not Assessed

Status

Listed in Schedule I of Wildlife (Protection) Act, 1972 and as Endangered on IUCN


Red List. Hence, statement 2 is incorrect.

www.insightsactivelearn.com 92
Total Marks : 200
Test-25 (Subject)
( INSTA Prelims Test Series 2021 )

Characteristics
Of the eight extant species of pangolin, the Indian Pangolin Manis crassicaudata and Chinese
Pangolin M. pentadactyla occur in India. Indian Pangolin is a large anteater covered dorsally
by 11-13 rows of scales. The adult male is about one-third larger than the female. A terminal
scale is also present on the ventral side of the tail of the Indian Pangolin, which is absent in
the Chinese Pangolin. Its sticky tongue, which is longer than its body, is specially adapted for
reaching and lapping up insects in deep crevices. To tear open the anthills or termite mounds,
it uses the powerful forelimbs that are armed with three disproportionately long claws. In
sharp contrast, the hind legs have tough soles and short, blunt nails on the five toes.

https://www.wwfindia.org/about_wwf/priority_species/threatened_species/indian_pangolin/

94. Consider the following statements regarding Legal Entity Identifier (LEI) code
1. LEI is a 20-digit unique code to identify parties to financial transactions worldwide.
2. Corporate borrowers having ₹ 50 crore and above from any bank to obtain Legal Entity
Identifier (LEI) registration.

Which of the statements given above is/are correct?


A. 1 only
B. 2 only
C. Both 1 and 2
D. Neither 1 nor 2

Correct Answer : A

Answer Justification :

Legal Entity Identifier (LEI) - It has been decided to require banks to make it
mandatory for corporate borrowers having aggregate fund-based and non-fund based
exposure of ₹ 5 crore and above from any bank to obtain Legal Entity Identifier (LEI)
registration and capture the same in the Central Repository of Information on Large

www.insightsactivelearn.com 93
Total Marks : 200
Test-25 (Subject)
( INSTA Prelims Test Series 2021 )

Credits (CRILC). Hence, statement 2 is incorrect.

This will facilitate assessment of aggregate borrowing by corporate groups, and monitoring of
the financial profile of an entity/group. This requirement will be implemented in a calibrated,
but time-bound manner. Necessary instructions will be issued by end-October 2017.

Introduction of Legal Entity Identifier for large corporate borrowers

The Legal Entity Identifier (LEI) code is conceived as a key measure to improve the quality and
accuracy of financial data systems for better risk management post the Global Financial Crisis.
LEI is a 20-digit unique code to identify parties to financial transactions worldwide.

https://rbi.org.in/scripts/FS_Notification.aspx?Id=11154&Mode=0&fn=2

95. Consider the following statements regarding North East Venture Fund (NEVF)
1. It is the first and the only dedicated Venture Fund for the North Eastern Region.
2. It was launched by NITI aayog.
3. The investment size with the Venture Fund Scheme ranges between Rs.25 lakhs and Rs. 10
crores.

Which of the statements given above are correct?


A. 1 and 2 only
B. 2 and 3 only
C. 1 and 3 only
D. 1, 2 and 3

Correct Answer : C

Answer Justification :

The North East Venture Fund (NEVF), which is the first and the only dedicated
Venture Fund for the North Eastern Region and was introduced by the Modi
government, is gaining popularity among Start-Ups and young entrepreneurs. The
Venture Fund Scheme launched by the Ministry of Development of North Eastern
Region (DoNER) intended to promote growth of business ventures and skill
development in the region. Hence, statement 2 is incorrect.

Giving an update on the scheme, Union Minister of State (Independent Charge) Development
of North Eastern Region (DoNER), MoS PMO, Personnel, Public Grievances, Pensions, Atomic
Energy and Space, Dr.Jitendra Singh said here today that the Ministry of DoNER had joined
with North Eastern Development Finance Corporation Ltd (NEDFi) to set up the North East
Venture Fund, the first and the only Venture Fund for Northeast with an initial corpus of Rs.
100 crores. The fund targets to invest in Start-Ups and unique business opportunities to
provide resources for new entrepreneurships. The main focus of North East Venture Fund
(NEVF), he said, is for mostly the enterprises involved in Food Processing, Healthcare,

www.insightsactivelearn.com 94
Total Marks : 200
Test-25 (Subject)
( INSTA Prelims Test Series 2021 )

Tourism, segregation of services, IT, etc.

The investment size with the Venture Fund Scheme ranges between Rs.25 lakhs and Rs. 10
crores with a long term horizon of five to ten years.

https://pib.gov.in/PressReleasePage.aspx?PRID=1687668

96. Consider the following statements regarding Petroleum and Explosives Safety Organization (PESO)
1. PESO’s major work is to administer the responsibilities delegated under the Explosives Act
1884 and Petroleum Act 1934.
2. It has been serving the nation as a nodal agency for regulating safety of hazardous substances
such as explosives, compressed gas and petroleum.

Which of the statements given above is/are correct?


A. 1 only
B. 2 only
C. Both 1 and 2
D. Neither 1 nor 2

Correct Answer : C

Answer Justification :

All the above statements are correct.

The Petroleum and Explosives Safety Organization (PESO), formerly known as


Department of Explosives, since its inception on 05/09/1898, has been serving the
nation as a nodal agency for regulating safety of hazardous substances such as
explosives, compressed gas and petroleum.

PESO’s major work is to administer the responsibilities delegated under the Explosives Act
1884 and Petroleum Act 1934 and the Rules made there under related to manufacture, import,
export, transport, possession, sale and use of Explosives, Petroleum products and Compressed
gases.

The organization has earned rare distinction as an institution of excellence in matters related
to safety in manufacturing/refining, storage, transportation, handling and use of hazardous

www.insightsactivelearn.com 95
Total Marks : 200
Test-25 (Subject)
( INSTA Prelims Test Series 2021 )

substances for over a century. Apart from the normal functions of enforcement of statutory
safety regulations to safeguard public safety, life, property and environment, the organization
has rendered meritorious voluntary services in examination and disposal of explosives,
improvised explosives devices till late eighties of the last century, some of which were of
national importance encountered during freedom struggle of the country, terrorist activities in
different regions in the country. Till early nineties of the last century, the officers of the
organization were carrying out anti-sabotage checks and duties related to VVIP security,
Airport Security etc.

https://peso.gov.in/web/about-peso

97. 'Vajra Prahar' was a joint military exercise conducted by India with

A. USA
B. United Kingdom
C. France
D. Russia

Correct Answer : A

Answer Justification :

Special forces of India and the US carried out a joint military exercise at Bakloh in Himachal
Pradesh this month to further enhance interoperability between the two sides, the defence
ministry said on Tuesday.

It was the 11th edition of the exercise 'Vajra Prahar' which is aimed at sharing best practices
and experiences in areas such as joint mission planning and operational tactics. Hence,
option (a) is correct.

India and the US carried out a two-day naval exercise in the eastern Indian Ocean Region on
March 28 ad 29.

The Indian Navy deployed its warship Shivalik and long-range maritime patrol aircraft P8I in
the 'PASSEX' exercise while the US Navy was represented by USS Theodore Roosevelt carrier
strike group.

https://economictimes.indiatimes.com/news/defence/special-forces-of-india-and-us-carry-out-mi
litary-drill-in-himachal-pradesh/articleshow/81776729.cms

98. Consider the following statements regarding Dindori Project on millets


1. It is supported by FAO.
2. It is an initiative to revive kodo millet and kutki (little millet) cultivation in Dindori district of

www.insightsactivelearn.com 96
Total Marks : 200
Test-25 (Subject)
( INSTA Prelims Test Series 2021 )

Madhya Pradesh.

Which of the statements given above is/are correct?


A. 1 only
B. 2 only
C. Both 1 and 2
D. Neither 1 nor 2

Correct Answer : B

Answer Justification :

Dindori Project

International Fund for Agricultural Development (IFAD) has supported an initiative to


revive kodo millet and kutki (little millet) cultivation in Dindori district of Madhya
Pradesh. Hence, statement 1 is incorrect.

The IFAD project was started in 2013-14, with women-farmers from 40 villages - mostly
from the Gonda and Baiga tribes - growing these two minor millets.

The identified farmers were supplied good-quality seeds and trained by Jawaharlal
Nehru Agricultural University in Jabalpur and the local Krishi Vigyan Kendra.

They were trained on field preparation, line-sowing and application of compost, zinc,
bavastin fungicide and other specific plant protection chemicals.

Further, a federation of the farmers’ self-help groups undertook procurement of the


produce and also its mechanical de-hulling.

[Mechanical de-hulling is the traditional time-consuming manual pounding process to


remove husk from the grain.]

The IFAD project has helped in meeting nutritional goals and reviving millet cultivation.

IFAD has supported an initiative to revive kodo and kutki cultivation in Dindori district of
Madhya Pradesh. The project was started in 2013-14, with 1,497 women-farmers from 40
villages – mostly from the Gonda and Baiga tribes – growing these two minor millets on 749
acres.

https://indianexpress.com/article/india/in-madhya-pradeshs-dindoi-a-un-backed-project-aims-to-

www.insightsactivelearn.com 97
Total Marks : 200
Test-25 (Subject)
( INSTA Prelims Test Series 2021 )

put-millets-7249608/

99. Consider the following statements regarding Mahatma Gandhi National Fellowship (MGNF)
1. MGNF is a Certificate Program in Public Policy and Management offered by IITs.
2. It is an opportunity for young, dynamic individuals to contribute to enhancing skill
development and promote economic development.

Which of the statements given above is/are correct?


A. 1 only
B. 2 only
C. Both 1 and 2
D. Neither 1 nor 2

Correct Answer : B

Answer Justification :

Mahatma Gandhi National Fellowship


(MGNF)
MGNF is a Certificate Program in Public Policy and Management offered by IIMs.
Hence, statement 1 is incorrect.

It has been designed at the initiative of the Ministry of Skill Development and
Entrepreneurship (MSDE), Government of India (GoI) and implemented in collaboration with
State Skill Development Missions (SSDMs).

The Mahatma Gandhi National Fellowship (MGNF) is an opportunity for young,


dynamic individuals to contribute to enhancing skill development and promote
economic development.

The two-year Fellowship seeks to combine classroom sessions (Academic Module) host IIMs
with an intensive field immersion at the district level (District Immersion) to create credible
plans and identify barriers in raising employment, economic output, and promote livelihoods in
rural areas.

https://www.iimb.ac.in/mgnf

100. Consider the following statements regarding Tholpavakoothu


1. It is the traditional shadow puppetry of Tamil Nadu.
2. The puppeteers perform the story of the Ramayana.

www.insightsactivelearn.com 98
Total Marks : 200
Test-25 (Subject)
( INSTA Prelims Test Series 2021 )

Which of the statements given above is/are correct?


A. 1 only
B. 2 only
C. Both 1 and 2
D. Neither 1 nor 2

Correct Answer : B

Answer Justification :

How Tholpavakoothu, the traditional puppetry of Kerala, emerged out of the shadows.

Tholpavakoothu is believed to be more than 350 years old.

Performed in the Bhadrakali temples, from January to June every year, the shadow
puppeteers of Kerala are busy performing the story of the Ramayana as part of the
rituals during temple festivals. Hence, statement 1 is incorrect.

Ramachandra Pulavar, son of the late guru KL Krishnankutty Pulavar, belongs to the 13th
generation in a family of puppeteers. For him, this season of plays had an auspicious start, he
has been awarded the Padma Shri. He is the first folk artiste from Kerala to be selected for
this honour.

https://www.thehindu.com/news/national/kerala/watch-how-tholpavakoothu-the-traditional-pup
petry-of-kerala-emerged-out-of-the-shadows/article34220624.ece

www.insightsactivelearn.com 99

You might also like